Download as pdf or txt
Download as pdf or txt
You are on page 1of 96

Ka

vi
ra
cl
as
se
s
de
lh
i
C
O
Class - IX
N
T Serial
No.
UNIT - 1
Page
No.

E 1. Motion 1

N 2. Force and Laws of Motion 49


T
S
i
lh
de
s
se
as
cl
ra
vi
Ka
ll
Ka
vi
ra
cl
as
se
Chapter

s
de
1

lh
i
10.0 EQUATIONS OF UNIFORMLY
1.0 MOTION ACCELERATED MOTION
2.0 TYPES OF MOTION 10.1 First Equation of Motion
2.1 Translational Motion 10.2 Second Equation of Motion
(or Translatory Motion) 10.3 Third Equation of Motion
2.2 Rotational Motion (Rotatory 10.4 Distance Covered by the Particle
Motion) in nth Second
2.3 Vibratory (or Oscillatory) Motion
11.0 FREE FALL (MOTION UNDER
3.0 CONCEPT OF A POINT GRAVITY)
OBJECT (OR PARTICLE)
12.0 GRAPHICAL REPRESENTATION
4.0 DESCRIBING MOTION OF MOTION
5.0 SCALAR AND VECTOR 12.1 Distance-Time Graph
12.2 Displacement-Time Graph
QUANTITIES
12.3 Speed-Time Graph
5.1 Scalar Quantity
12.4 Velocity-Time Graph
5.2 Vector Quantity 12.5 Acceleration-Time Graph
5.3 Difference between Scalar &
Vector Quantities 13.0 EQUATIONS OF MOTION BY
GRAPHICAL METHOD
6.0 DISTANCE AND DISPLACEMENT 13.1 Derivation of v = u + at
6.1 Distance 1 2
6.2 Displacement 13.2 Derivation of s = ut + at
2
6.3 Comparison between
13.3 Derivation of v2 = u2 + 2as
Distance and Displacement
14.0 CIRCULAR MOTION
7.0 SPEED AND VELOCITY 14.1 Difference between Uniform
7.1 Speed Linear Motion and Uniform
7.2 Velocity Circular Motion
7.3 Comparison between Speed 14.2 Radian
and Velocity 14.3 Angular Displacement and
Unit One

Angular Velocity
8.0 UNIFORM AND NON UNIFORM 14.4 Relation between Linear Speed
MOTION and Angular Velocity
8.1 Uniform Motion 14.5 Centripetal Acceleration
8.2 Non-uniform Motion
EXERCISE 1 (ELEMENTARY)
9.0 ACCELERATION
EXERCISE 2 (SEASONED)
9.1 Uniform Acceleration
9.2 Non-uniform Acceleration or EXERCISE 3 (CBSE PATTERN)
Variable Acceleration EXERCISE4 (COMPETITIVE ASSESSMENT)
i
lh
de
s
se
as
cl
ra
vi
Ka
kk
Ka
vi
Moti on

ra
MOTION (MOT)

cl
as
(MOT-1)

se
s
1.0 MOTION

de
If we look around us, we find that there are number of objects which are in motion.
An object is said to be in motion if it changes its position with the passage of time.

lh
Now observe the following bodies or objects to understand the meaning of the term "motion". Cars,

i
cycles, motorcycles, scooters, buses, rickshaws, trucks, etc. running on the road, birds flying in the
sky, fish swimming in water, all these objects are in motion. Very small objects like atoms and
molecules and very large objects like planets, stars and galaxies are also in motion.
Thus, all objects ranging from the smallest atom to the largest galaxy are in continuous motion.

2.0 TYPES OF MOTION


2.1 Translational Motion (or Translatory Motion)
Motion of a body in which all the points in the body follow parallel paths is called ‘translational
motion’. It is a motion in which the orientation of an object remains the same throughout the
journey . The path of a translatory motion can be straight or curved (see fig.1).

(a) A bo ok m oved a long a straight path w ith out cha nging its o rientat ion

(b) A bo ok m oved a long a curved path w ith out changing it s o rientation

Fig.1 Translational motion

On the basis of the path travelled by an object, the translational motion can classified as

(1) Rectilinear motion - If an object moves in a straight line, A


its motion is called rectilinear motion or one dimensional
motion. Motion of car along a straight path, motion of a piston B C
D\Allen-Junior wing\Physics\IX\Unit-1\01-Motion\2019-20

in the cylinder are examples of rectilinear motion.


x x x
(2) Curvilinear motion - If an object moves along a curved
path without change in its orientation, its motion is called
A'
curvilinear motion. Motion of a car along a curved or circular B' C'
path, motion of an athlete on a circular track are examples
of curvilinear motion.
Fig. 2 Translational motion
The particles of the object shown
cover same distance in a given time.
1
Ka
vi
Class IX - Physics

ra
2.2 Rotational Motion (Rotatory Motion)

cl
Motion of a body turning about an axis is called rotational motion. In other words, ‘a motion in

as
which an object spins about a fixed axis is called rotational motion’. It is a motion in which the

se
orientation of an object continuously changes throughout the motion. The path of an object in a

s
rotational motion is always circular.
Some examples of rotational motion are :

de
(1) The Earth’s spin on its axis. (2) Motion of a fan or motor.

lh
(3) Motion of blades of windmill. (4) Motion of a spinning top.

i
(5) Motion of a grinding stone.

(a) Motion of a ceiling fan (b) Motion of Earth about its axis (c) Motion of a spinning top

Fig.3 Some examples of rotational motion

In rotational motion, the particles of the object move through


A y
the unequal distances in a given time depending on their
B
location in the object (see fig.4). A'
x
The particle which is located near the axis of rotation, covers B'
less distance as compared to the particle that is located far
away from the axis.
l In translational motion at any instant of time every particle of
the body has the same velocity while in rotational motion at A x is o f
any instant of time particles of the body have different ro tatio n
velocities depending on their position from the axis of rotation. y>x
F ig.4 R o tat io n al m o tion : Par ticle s
l In rotation of a body about a fixed axis, every particle of the co ver u n eq ual distance s in a given
body moves in a circle, which lies in a plane perpendicular to tim e.
the axis and has its centre on the axis.

2.3 Vibratory (or Oscillatory) Motion


A body has vibratory motion if it moves to and fro about a fixed point.
Examples : (i) Motion of the pendulum of a wall clock.
(ii) Motion of a simple pendulum.
Bob
Fig. 5 Vibratory motion
3.0 CONCEPT OF A POINT OBJECT (OR PARTICLE)
Point object - An extended object can be treated as a point object when the distance travelled
by the object is much greater than its own size.
D\Allen-Junior wing\Physics\IX\Unit-1\01-Motion\2019-20

A point object (or particle) is one, which has no linear dimensions but possesses mass.
Examples - (i) Study of motion of a train travelling from Jaipur to New Delhi. (ii) Revolution
of earth around the sun for one complete revolution.

4.0 DESCRIBING MOTION


When a tree is observed by an observer A sitting on a bench, the tree is at rest. This is because
position of the tree is not changing with respect to the observer A.
Now, when the same tree is observed by an observer B sitting in a superfast train moving with
a velocity v, then the tree is moving with respect to the observer because the position of tree is
changing with respect to the observer B.
2
Ka
vi
Moti on

ra
cl
as
se
s
de
lh
i
Fig. 6 Describing motion

Rest and motion are relative terms - There is nothing like absolute rest. This means that
an object can be at rest and also in motion at the same time i.e. all objects, which are stationary
on earth, are said to be at rest with respect to each other, but with respect to the sun they are making
revolutions. In order to study motion, therefore, we have to choose a fixed position or point with
respect to which the motion has to be studied. Such a point or fixed position is called a reference
point or the origin.

Illustration 1. Discuss whether the walls of your classroom are at rest or in motion.
Solution The walls of our classroom are at rest with respect to the ground or earth. But, they
are in motion with respect to an object or an observer outside the earth. This is because
the earth is moving about its own axis as well as it is revolving around the sun. Thus,
the state of rest and motion are not absolute, they are relative terms.

Illustration 2. In fig.7, motion a frying pan used in kitchen is shown. Is the motion of the frying pan
a translational motion ? Can it be considered as rotational motion ? Explain.

Fig.7

Solution The motion of frying pan shown in fig.7 cannot be considered as translational motion
though it is moving along a curved path. This is because its orientation is changing during
its journey. Also, the motion of frying pan cannot be considered as rotational motion
D\Allen-Junior wing\Physics\IX\Unit-1\01-Motion\2019-20

though it is spinning. This is because, rotation means spinning of an object about a


fixed axis. Here, the flask is not spinning about a fixed axis. This type of motion is ‘a
combination of translational motion and rotational motion’.

3
Ka
vi
Class IX - Physics

ra
Motion of a car or cycle wheels is a combination of translational and rotational motion (see fig.8).

cl
l

as
se
Tran s la tio n al
m o tio n

s
de
lh
i
R o tat io na l m ot io n
Fig.8 Motion of a lylle wheel is a lombination of translational and
rotational motion. Motion of a wheel is also lalled ‘rolling motion’.

Illustration 3. Give two examples where we feel the presence of motion through indirect evidences.
Solution. (i) Trees, electric poles appear to move opposite to the direction of a moving train to the
observer sitting within the train.
(ii) Everyday we see that the Sun rises in the east and sets in the west. This is an
indirect evidence of the fact that the Earth spins about its own axis from west to east.
Illustration 4. A person is sitting in a moving train. Examine and define its state.
Solution. The person sitting inside the moving train is in motion when the frame of reference is
taken as the ground (Earth). But when we consider the moving train as the frame of
reference, the person is in the state of rest. So, to an observer standing on the ground,
the person appears to be moving and to an observer sitting inside the same moving
train, the person appears to be at rest.
Illustration 5. Give one example each of the following types of motion:
(a) Rectilinear motion (b) Curvilinear motion
(c) Oscillatory motion (d) Vibratory motion
(e) Combination of rotational and translational motion
Solution. (a) A stone falling vertically downward when released from some height.
(b) Motion of a javelin or shot put thrown by an athlete
(c) Swinging of pendulum bob
(d) Flapping of mosquito’s wings
(e) Motion of wheel of a running bicycle.
D\Allen-Junior wing\Physics\IX\Unit-1\01-Motion\2019-20

4
Ka
vi
Moti on

ra
cl
as
Objective Questions

se
1. A body whose position with respect to surrounding does not change, is said to be in a state of :

s
(A) Rest (B) Motion (C) Vibration (D) Oscillation

de
2. Consider the following statements :

lh
I : Train is at rest with respect to observer P ;

i
II : Train is in motion with respect to observer Q
(A) I and II can be true only at different times (B) I and II can be true at the same time
(C) If I is true, II can never be true (D) None of these
3. Motion along a straight line is called
(A) Rectilinear motion (B) Circular motion
(C) Oscillatory motion (D) Parabolic motion
4. Which of the following statement is correct
(A) Only absolute rest is possible
(B) Only absolute motion is possible
(C) Both absolute rest and absolute motion are possible
(D) Neither absolute rest nor absolute motion is possible
5. Which two fundamental properties are used to describe motion?
(A) Mass & distance (B) Length & time (C) Speed & mass (D) Distance & speed
Subjective Questions
6. Is it possible that the train in which you are sitting appears to move while it is at rest? Explain.
7. You are moving towards your home. Is your home at rest with respect to you or is it moving with
respect to you? Explain.
8. Is there any object in this universe which is at rest? Support your answer with explanation.
9. Two runners are running along parallel tracks in the same direction such that they cover equal
distance in equal time. Are they (i) At rest with respect to ground? (ii) Moving with respect
to each other?
10. Explain translational and rotational motion along with examples.
D\Allen-Junior wing\Physics\IX\Unit-1\01-Motion\2019-20

5
Ka
vi
Class IX - Physics

ra
MOTION

cl
as
(MOT-2)

se
s
5.0 SCALAR AND VECTOR QUANTITIES

de
5.1 Scalar Quantity

lh
A physical quantity that is defined by its magnitude only is called a scalar quantity.

i
Examples - Mass, time, distance, speed, work, power, energy, electric charge, volume, density,
pressure, electric potential, temperature, etc.
Scalar quantities follow the algebraic (scalar) laws of addition.

5.2 Vector Quantity


A physical quantity that is defined by its magnitude as well as direction is called a vector quantity.
Examples - Velocity, acceleration, force, displacement, momentum, weight, area, torque, electric
field, magnetic field, etc.
Vector quantities follow the vector laws of addition.

5.3 Difference between Scalar & Vector Quantities


Scalar quantities Vector quantities
These quantities are completely These quantities are completely specified by their
1
specified by their magnitude only. magnitude as well as direction.
These quantities change by change in These quantities change by change in either their
2
their magnitude only. magnitude or direction or both.
These quantities are added or These quantities are added or subtracted by laws
3
subtracted by laws of ordinary algebra. of vector addition.

6.0 DISTANCE AND DISPLACEMENT


6.1 Distance
The length of the actual path between the initial and the final position of a moving object in the
given time interval is known as the distance travelled by the object.
Distance = Length of path I (ACB) I C
Distance is a scalar quantity. It is always taken positive.
II
Distance is measured by odometer in vehicles A B
Units - In SI system - metre (m). Fig. 9 Concept of distance
In CGS system - centimetre (cm). and displacement

6.2 Displacement
D\Allen-Junior wing\Physics\IX\Unit-1\01-Motion\2019-20

The shortest distance between the initial position and the final position of a moving object in the
given interval of time is known as the displacement of the object.
Displacement = Length of path II (AB)
Displacement of an object may also be defined as the change in position of the object in a particular
direction. That is,
Displacement of an object = Final position – Initial position
of the object = xƒ – xi.

6
Ka
vi
Moti on

ra
cl
During motion, displacement of an object may be zero but the

as
distance travelled by the object is never zero.
+ y-axis

se
Distance travelled by an object is either equal to or greater than
(Vertical direction)

s
the magnitude of displacement of the object.
x-axis
– +

de
Displacement is a vector quantity. (Horizontal direction)

lh
Displacement can be positive, negative or zero.
Fig. 10 Sign convention

i
Units - In SI system - metre (m)
for displacement
In CGS system - centimetre (cm)

6.3 Comparison between Distance and Displacement

Distance Displacement
1 It is defined as the length of the actual It is the shortest distance between two
path traversed by a body. points which the body moves.
2 It is scalar quantity. It is a vector quantity.
3 It is always positive. It can be negative, positive or zero.
4 Distance can be equal to or greater Displacement can be equal to or less than
than displacement. distance.
5 Distance travelled is not a unique Displacement is a unique path between
path between two points. two points.
6 The distance between two points Displacement between two points does not
gives full information of the type of give full information of the type of path
path followed by the body. followed by the body.
7 Distance never decreases with time. Displacement can decrease with time. For
For a moving body, it is never zero. a moving body, it can be zero.
8 Distance in SI unit is measured in Displacement in SI unit is measured in
meter. meter.

Illustration 1. A honeybee leaves the hive and travels 2 m before returning. Is the displacement for
the trip the same as the distance travelled ? If not, why not ?

H one y bee

H ive
D\Allen-Junior wing\Physics\IX\Unit-1\01-Motion\2019-20

Fig.11
Solution No, the displacement and the distance are not same. This is because the displacement
is the change of position of object in motion while distance is length of path travelled
by it.
Here, the distance travelled = 2 m
While, the displacement = 0, because the position of honey bee is not changed.

7
Ka
vi
Class IX - Physics

ra
Illustration 2. Motion of a particle is shown below on a number line. Find the displacement from (a)

cl
A to B (b) B to C (c) overall journey. Also, find distance for overall journey.

as
F in al p o sit io n In itial p o sition

se
C A B

s
Interm ediate p os ition

de
–8 –6 –4 –2 0 2 4 6 8 (in m eters )

lh
Fig. 12

i
Solution (a) Displacement from A to B = xƒ – xi = (6) – (0) = +6 m
(b) Displacement from B to C = xƒ – xi = (–6) – (6) = –12 m
(c) Displacement of overall journey (i.e. A to B, B to C)
= xƒ – xi = (–6) – (0) = –6 m
Here, distance can also be found by adding positive values of displacement AB &
displacement BC. i.e.,Distance travelled during overall journey
= AB + BC = 6 + 12 = 18 m
Here Distance > |Displacement|
Whenever a particle changes its direction, distance is greater than displacement.

Illustration 3. A body falls from a height of 3 m. Find displacement and


distance. 3m

Solution Displacement = –3 m
Distance = 3 m
Fig. 13

Illustration 4. A particle moves along a circular path as shown in figure.


Find distance travelled and displacement.
1 R R
Solution Distance travelled = × (circumference of the circle) A B
2
Fig. 14
1
= × (2pR) = pR
2
Displacement = diameter AB = 2R

Illustration 5. In the fig.15, a car moves on the road from the 20 km mark (its initial position) to the
100 km mark. After that, it reverses and moves back to the 50 km mark (its final position).
Find the displacement and distance travelled by the car.

A B C
D\Allen-Junior wing\Physics\IX\Unit-1\01-Motion\2019-20

20 k m 50 k m 100 k m

Fig.15
Solution Given, initial position, xi = + 20 km ; final position, xf = +50 km
Displacement = xf – xi = (+50) – (+20) = + 30 km
Now, distance travelled by car from A to C, AC = 100 – 20 = 80 km
Distance travelled by car from C to B, BC = 100 – 50 = 50 km
Total distance travelled by car = AB +BC = 80 + 50 = 130 km

8
Ka
vi
Moti on

ra
cl
as
Objective Questions

se
s
1. For motion on a straight line path with constant velocity, the ratio of the magnitude of the dis-
placement to the distance covered is

de
(A) = 1 (B) ³ 1 (C) £ 1 (D) < 1

lh
i
2. In 1.0 s, a particle goes from point A to point B, moving in a semicircle of radius 1.0 m (see figure).
The magnitude of the displacement is A
(A) 3.14 m
(B) 2.0 m

1.
(C) 1.0 m

0m
(D) zero B

3. Which of the following is not a characteristic of displacement :


(A) It is always positive
(B) It has both magnitude and direction
(C) It can be zero
(D) Its magnitude may be less than or equal to the actual path length of the object

4. A body is moving on a square track of side 1 m. It completes one revolution in 40 seconds. Calculate
its displacement in 4 minutes :
(A) 2m (B) 3m (C) 0 m (D) 24 m

5. Consider the motion of the tip of the minute hand of a clock. In one hour
(A) the displacement is zero
(B) the distance covered is zero
(C) the displacement is 2p × (length of minute hand)
(D) None of these

Subjective Questions

6. An object has moved through a distance. Can it have zero displacement?

7. An athlete completes one round of a circular track of diameter 200 m in 40 s. What will be the
distance covered and also the displacement at the end of 2 min 20 s?

8. In a long distance race, the athletes were expected to take four rounds of the track such that the
line of finish was same as the line of start. Suppose the length of the track was 200 m.
(i) What is the total distance to be covered by the athletes ?
(ii) What is the displacement of the athletes when they touch the finish line?
D\Allen-Junior wing\Physics\IX\Unit-1\01-Motion\2019-20

(iii)Is the displacement of an athlete and the distance moved by him at the end of the race equal?

9. An athlete runs along a circular track of radius 50 m. Find the distance travelled and the displacement
3
of the athelete when he covers th of the circle.
4

10. An object moves 60 m due east and then 80 m due north. Find the distance travelled and the
magnitude of displacement.

9
Ka
vi
Class IX - Physics

ra
MOTION

cl
as
(MOT-3)

se
s
7.0 SPEED AND VELOCITY

de
7.1 Speed

lh
Speed of a body is the distance travelled by the body per unit time. The rate of change of distance
is called speed.

i
distance travelled
Speed =
time taken
If a body covers a distance s in time t then speed,
s
v=
t
Unit - In SI system : m/s or ms–1
In CGS system : cm/s or cms–1
A commonly used unit of speed is km/h or kmh–1.
Speed is a scalar quantity, because it has magnitude but no direction. Speed is always taken
positive.
Uniform speed - When a body covers equal distances in equal intervals of time, the body is
to be moving with a uniform speed or constant speed.
Examples (i) A train running with a speed of 120 km/h. (ii) An aeroplane flying with a speed
of 600 km/h.
Non-uniform speed - When a body covers unequal distances in equal intervals of time, the body
is said to be moving with non-uniform speed or variable speed.
Examples - (i) A car running on a busy road. (ii) An aeroplane landing on a runway.
Average speed - The average speed of the body in a given time interval is defined as the total
distance travelled, divided by total time taken.
Total distance travelled
Average speed =
Total time taken

Instantaneous speed - The speed of a body at any particular instant of time during its motion
is called the instantaneous speed of the body. It is measured by speedometer in vehicles.

7.2 Velocity
The velocity of a body is the displacement of a body per unit time. y-a x is (V e r t ica l d ire c t io n )
+
Displacement
Velocity =
Time x -a x is
– + (H o r izo n ta l d ire ct io n )
D\Allen-Junior wing\Physics\IX\Unit-1\01-Motion\2019-20

The displacement covered by a body per unit time or the speed of a



body in specified direction is called velocity.
Velocity is a vector quantity. It can be positive, negative or zero. Fig. 16 Sign convention for velocity

Unit - In SI system : m/s or ms–1


In CGS system : cm/s or cms–1

Uniform velocity - When a body covers equal displacements in equal intervals of time in a
particular direction, the body is said to be moving with a uniform velocity.

10
Ka
vi
Moti on

ra
Conditions for uniform velocity are A B C D m o tio n

cl
(i) The body must cover equal displacements in equal

as
5m 5m 5m
intervals of time.
1s 1s 1s

se
(ii) The direction of motion of the body should not change.

s
Example - A train running towards south with a speed of
120 km/h. Fig. 17 Body moving with

de
uniform velocity

lh
Non-uniform velocity/variable velocity - When a body covers unequal displacements in equal

i
intervals of time, the body is said to be moving with variable velocity.
When a body covers equal distances in equal intervals of time, but its direction changes, then the
body is said to be moving with variable velocity.
A
Example - In circular motion, a particle may have constant speed but
5m
its direction changes continuously thus, its velocity is non-uniform. 1s 1 s 5m
Conditions for variable velocity are (i) It should cover unequal
displacements in equal intervals of time. (ii)It should cover D B
equal distances in equal intervals of time but its direction
must change. 1s 1s
5m 5m
Examples - (i) A car running towards north on a busy road has
a variable velocity as the displacement covered by it per unit time C
changes with change
Fig. 18 Body moving with variable velocity
in the road condition.
(ii)The blades of a rotating ceiling fan, a person running around a circular track with constant speed
etc. are the examples of variable velocity.
Average velocity - Total displacement of a particle divided by total time taken is called average
velocity.
A B C D
m ot ion
Total displacement 5m 3m 7m
Average velocity =
Total time taken 1s 1s 1s

x 2 - x1
Vav = t - t Fig. 19 Body moving with non-uniform velocity
2 1

l Average speed is always greater than or equal to magnitude of average velocity. Average speed
is equal to average velocity when particle moves in a straight line without change in direction.
Instantaneous velocity is the velocity of a body at any particular instant of time during its motion.

7.3 Comparison between Speed and Velocity

Speed Velocity
1 It is defined as the rate of change of It is defined as the rate of change of
distance. displacement.
D\Allen-Junior wing\Physics\IX\Unit-1\01-Motion\2019-20

2 It is a scalar quantity. It is a vector quantity.


3 It is always positive. It can be negative, positive or zero.
4 Speed is velocity without direction. Velocity is directed speed.
5 Speed in SI unit is measured in ms–1. Velocity in SI unit is measured in ms –1.
ACTIVE PHYSICS - On any cloudy day or night, sometimes we see frequent lightning and hear
the sound of thunder. The sound of thunder takes some time to reach us after we see the lightning.
This is because light travels with very high speed (= 3 × 108 m/s) while sound travels with much
lower speed (= 346 m/s).

11
Ka
vi
Class IX - Physics

ra
To measure the distance (s) to the nearest point of thunder,

cl
we first measure the time interval (t) between the lightning and

as
Cloud
thunder as observed by us using a stop watch.

se
Lightning
Thus, distance = speed of sound × time, or s = v × t = 346 × t

s
For example, if the time interval is 3 s then,

de
s = 346 × 3 = 1038m

lh
Fig.20

i
8.0 UNIFORM AND NON-UNIFORM MOTION
A moving body may cover equal distances in equal intervals of time or different distances in equal
intervals of time. On the basis of above assumption, the motion of a body can be classified as
uniform motion and non-uniform motion.
60
8.1 Uniform Motion
50
Time (in second) 0 1 2 3 4 5 6
40

D ista n c e
Distance covered (in metre) 0 10 20 30 40 50 60
30
When a body covers equal distances in equal intervals of time,
20
however small may be the time intervals, in a particular direction,
10
the body is said to describe a uniform motion.
0 1 2 3 4 5 6
l Uniform motion always takes place in a straight line. Fig. 21 Distance-time
T im e graph
for uniform motion
Examples of uniform motion are
(i) An aeroplane flying at a speed of 600 km/h along north.
(ii)A train running at a speed of 120 km/h along east. (iii) Light energy travelling at a speed of
3 × 108 m/s in vacuum.
16
8.2 Non-uniform Motion
Time (in second) 0 1 2 3 4
D ista nc e

9
Distance (in metre) 0 1 4 9 16
4
1
0 1 2 3 4 5 6
T im e
Fig. 22 Distance-time graph for non-uniform motion
When a body covers unequal distances in equal intervals of time, the body is said to be moving
with a non-uniform motion.
l Any motion along a curved path is always non-uniform motion. Also, any motion in which particle
changes its direction is also non-uniform motion.
Examples of non-uniform motion are
(i) An aeroplane running on a runway before taking off.
(ii) A freely falling stone under the action of gravity.
(iii) When the brakes are applied to a moving car.
D\Allen-Junior wing\Physics\IX\Unit-1\01-Motion\2019-20

(iv) A fan rotating with constant speed is also a non-uniform motion.

9.0 ACCELERATION
In uniform motion, the velocity remains constant with time. Thus, the change in velocity for any
time interval is zero. But, in non-uniform motion, velocity changes with time. Thus, the change
in velocity for any time interval has a non-zero value.
In non-uniform motion, a new physical quantity called 'acceleration' is used.
The rate of change of velocity of a moving body with time is called acceleration.

12
Ka
vi
Moti on

ra
+ y-a xis (Ver tical

cl
Change in velocity
Acceleration = direct ion)

as
Time taken for change
– x-a xis (Horizontal
+

se
But change in velocity = final velocity – initial velocity. direction)

s

Final velocity - Inital velocity v-u

de
Acceleration = a= Fig. 23 Sign convention for acceleration
Time taken for change t

lh
If body moves with uniform velocity, then v = u and then acceleration is zero i.e. a = 0.

i
l Acceleration is a vector quantity. It can be negative, positive or zero.
Unit of acceleration- In SI system : m/s2 or ms–2
In CGS system : cm/s2 or cms–2
If the velocity of an object increases with time, such a motion is called 'accelerated motion'. In
such motion, acceleration 'a' is considered positive for numerical problems.
Example - An object starts from rest and its velocity goes on increasing with time.
If velocity of an object decreases with time, such a motion is called 'retarded motion'.
In such a motion, acceleration is called 'retardation' or 'deceleration' and it is considered negative
for numerical problems.
Example - A vehicle in motion is stopped by applying brakes.
9.1 Uniform Acceleration A B C D
When a body moving in a straight line undergoes equal m otion
changes of velocity in equal intervals of time, the body is said 1s 1s 1s
to be moving with a uniform acceleration. Also, uniform
acceleration means an acceleration having a constant 2m /s 4m /s 6m /s 8m /s
magnitude and a constant direction.
Examples - (i) Motion of a freely falling body. Fig.24 Uniformly accelerating body
(ii) Motion of a ball rolling down on an inclined plane.
9.2 Non-uniform Acceleration or Variable Acceleration A B C D
When a body undergoes unequal changes of velocity in equal m o tio n
1s 1s 1s
intervals of time, the body is said to be moving with non-
uniform acceleration.
2 m /s 5 m /s 1 1m / s 1 3m / s
Examples - Fig. 25 Non-uniformly accelerating body
(i) The motion of a bus leaving or entering the bus stop.
(ii) A car moving on a busy road has non-uniform acceleration.

Illustration 1. A car travels first half distance with a uniform speed u and next half distance with a
uniform speed v. Find its average speed.
d d
Solution Total distance = + =d [See fig.26]
2 2 d

Total time = t1 + t2 = t
A d/2 d/2 B
D\Allen-Junior wing\Physics\IX\Unit-1\01-Motion\2019-20

d/2 é sù t1 t2
\ t1 =
u
...(i) êë t = v úû u v

d/2
t2 = ...(ii) Fig.26
v
d
Vav = . Putting the value of equation (i) and (ii),
t
d d 2 2uv
Vav = = = \ Vav =
d/2 d/2 d æ 1 1ö v+u u+ v
+ çè + ÷ø
u v 2 u v uv
13
Ka
vi
Class IX - Physics

ra
Illustration 2. A car travels first half time with a uniform speed u and next half time with a uniform

cl
speed v. Find its average speed.

as
t t

se
Solution d1 = u ´ , d2 = v ´ [See fig.27]
2 2

s
t
t t t/2 t/2

de
Total distance d = d1 + d 2 = u ´ + v ´ A B
2 2

lh
d1 d2
ut vt t
+ = (u + v)

i
d= u v
2 2 2
Total time = t Fig. 27
Total distance
Average speed =
Total time taken
t
(u + v) u+ v
Vav = 2 \ Vav =
t 2

Illustration 3. A particle moves along a path as shown in figure. The time taken during the journey
is 2 seconds. Find the average speed and average velocity during the journey.
Solution Total distance travelled,
s = AB + BL = 6 + 8 = 14 m Fin al
C
p osit ion
s 14 t
Average speed, Vav = = = 7m / s m en
t 2 ce 8m
la
isp
Here, displacement rs = AC = AB 2 + BC 2 d
In itial A B
= 2 2 p o sit io n 6m
(6) + (8) = 100 = 10 m
ur r
s 10
\ Average velocity, V av = = = 5 m/s Fig. 28
t 2
Here, average speed is greater than average velocity because the direction of particle
changes during motion.

Illustration 4. In your everyday life, you come across a range of motions in which
(a) Acceleration is in the direction of motion.
(b) Acceleration is against the direction of motion.
(c) Acceleration is uniform.
(d) Acceleration is non-uniform.
Identify one example each of the above types of motion.
Solution (a) While increasing the speed of vehicle using the accelerator, the acceleration is in
the direction of motion.
D\Allen-Junior wing\Physics\IX\Unit-1\01-Motion\2019-20

(b) While applying brakes of a vehicle, its speed decreases with time. Here, the
acceleration is against the direction of motion.
(c) Motion of a particle under gravity has a uniform acceleration
(g = 9.8 m/s2, vertically downwards ).
(d) Motion of a car in a crowded traffic has non-uniform acceleration as
its speed varies (increases or decreases) as per the need.

14
Ka
vi
Moti on

ra
Illustration 5. An object is sliding down on an inclined plane. The velocity changes at a constant rate

cl
from 10 cm/s to 15 cm/s in 2 seconds. What is its acceleration?

as
Solution. The situation is shown in figure 29. Let us take BA as the positive direction. The velocity

se
at t=0 is u = +10 cm/s and that at t = 2s is v = +15 cm/s.

s
v-u 15 cm / s - 10 cm / s

de
Thus, a = = 2 sec
t

lh
i
5 cm / sec
= = 2.5 cm/s2
2 sec
The acceleration is positive, which means it is in the direction BA. Fig. 29

Objective Questions
1. The position of a particle going along a straight line is x1 = 50m at 10.30 a.m. & x2 = 55m at 10.35
a.m. The velocity of the particle is :
(A) 2m/min (B) 5m/min (C) 1m/min (D) 0.5m/min

2. A body travels half the distance with speed of 20 m/s and the other half with speed of 30 m/s. The
average velocity of the body during whole journey is :
(A) zero (B) 24 m/s (C) 23 m/s (D) None of these

3. A 50 m long train passes over a bridge at a speed of 30 km/h. If it takes 36 seconds to cross the
bridge, the length of the bridge is :
(A) 100 m (B) 200 m (C) 250 m (D) 300 m

4. A car increases its speed uniformly from 10 m/sec to 60 m/sec in 10 sec. its acceleration is :
(A) 7m/s2 (B) 5 m/s2 (C) 6 m/s2 (D) None

5. The average velocity of a body is equal to the mean of its initial and final velocities. The acceleration
of the body is :
(A) uniform (B) variable (C) uniformly variable (D) zero

Subjective Questions
6. Two buses depart from Jaipur, one going to Kota and one to Delhi. Each bus travels at a speed
of 30 m/s. Do they have equal velocities? Explain.

7. One of the following statements is incorrect. (a) The car travelled around the track at a constant
velocity. (b) The car travelled around the track at a constant speed. Which statement is incorrect
and why ?
D\Allen-Junior wing\Physics\IX\Unit-1\01-Motion\2019-20

8. At a given instant of time, a car and a truck are travelling side by side in adjacent lanes of a highway.
The car has a greater velocity than the truck. Does the car necessarily have a greater acceleration?
Explain.

9. The average velocity for a trip has a positive value. Is it possible for the instantaneous velocity at
any point during the trip to have a negative value? Justify your answer.

10. The direction in which an object moves is given by the direction of velocity of the object and not by
the direction of acceleration. Give an example to justify this statement.

15
Ka
vi
Class IX - Physics

ra
MOTION

cl
as
(MOT-4)

se
s
10.0 EQUATIONS OF UNIFORMLY ACCELERATED MOTION

de
When an object moves with a uniform acceleration, its motion is called 'uniformly accelerated

lh
motion'.

i
v+u
In case of uniformly accelerated motion, the average velocity is given by Vav = ,
2
where v = final velocity, u = initial velocity
These equations give relationship between initial velocity, final velocity, time taken, acceleration
and distance travelled by the body.

10.1 First Equation of Motion


A body having an initial velocity 'u' acted upon by a uniform acceleration 'a' for time 't' such that
final velocity of the body is 'v'.
Change in velocity Final velocity – Initial velocity
Acceleration = =
Time taken Time taken
v-u
a= , or v – u = at , or v = u + at
t

10.2 Second Equation of Motion


It gives the distance travelled by a body in time 't'.
A body having an initial velocity 'u' acted upon by a uniform acceleration 'a' for time 't' such that
final velocity of the body is 'v' and the distance covered is 's'.
v+u
Vav =
2
Distance covered = average velocity× time taken
æ v + uö
s = çè ÷ ×t ... (1)
2 ø
but v = u + at (from first equation of motion) ... (2)
æ u + at + u ö æ 2u at ö æ at ö
Using (1) & (2), we get,s = ç ÷ × t = çè + ÷ × t =çu + ÷ t
è 2 ø 2 2ø è 2ø

1 2
or s = ut + at
2

10.3 Third Equation of Motion


A body having an initial velocity 'u' moving with a uniform acceleration 'a' for time 't' such that
D\Allen-Junior wing\Physics\IX\Unit-1\01-Motion\2019-20

final velocity is 'v' and the distance covered is 's'.


v+u
Vav =
2
Distance covered = average velocity × time taken
æ v + uö
s = çè ÷ ×t ... (1)
2 ø
Now, v = u + at, or v – u = at

16
Ka
vi
Moti on

ra
cl
æ v - uö
or t = çè ÷ ... (2)
a ø

as
se
From (1) & (2), we get,

s
æ v + uö æ v - uö v 2 - u2
s = çè ÷ø çè ÷ø =

de
2 a 2a

lh
or 2as = v2 – u2 , or v 2 = u 2 + 2as

i
10.4 Distance Covered by the Particle in nth Second
Let Sn and Sn–1 be the displacement of a particle in n and (n – 1) seconds respectively. Then, the
displacement of the particle in nth second is given by
Snth = Sn – Sn–1 ...(1)
1 2
We know, S = ut + at ...(2)
2
When t = n, then from eqn. (2), we have
1 2
Sn = un + an ...(3)
2
When t = (n –1), then from eqn. (2) we have
1
Sn–1 = u(n – 1) + a(n – 1)2 ...(4)
2
Substituting the value of eqns. (3) and (4) in eqn. (1), we get

1 2 é 1 2ù
Snth = un + an – ê u ( n - 1) + a ( n - 1) ú
2 ë 2 û

1 a
=u– a + an or Snth = u + (2n – 1)
2 2

l Conversion from kmh–1 to ms –1 and vice-versa


1km 1000 m 5
= = m/s
h 60×60 s 18

5
Velocity in m/s = × velolity in km/h
18

×5
18 ®
¾¾¾
¬¾¾ ms –1
Km h–1 18 ¾
×
D\Allen-Junior wing\Physics\IX\Unit-1\01-Motion\2019-20

11.0 FREE FALL (MOTION UNDER GRAVITY)


Till 1600 AD, the teachings of the Greek philosopher Aristotle (384–322 BC) had held that heavier
objects fall faster than lighter ones. The Italian physicist Galileo Galilei gave the present day ideas
of falling objects. Now, it is an established fact that, in the absence of air resistance, all objects
dropped near the Earth’s surface fall with the same constant acceleration under the influence of the
Earth’s gravity.
Free fall is the motion of an object subject only to the influence of gravity. An object is in free fall as
soon as it is dropped from rest, thrown downward or thrown upward.

17
Ka
vi
Class IX - Physics

ra
Acceleration due to gravity - The constant acceleration of a freely falling body is called the

cl
acceleration due to gravity.

as
l The acceleration due to gravity is the acceleration of an object in free fall that results from the

se
influence of Earth’s gravity. Its magnitude is denoted with the letter g. The value of g on the surface

s
of Earth is nearly 9.8 m/s2. In C.G.S. system, g = 980 cm/s 2 ; in F.P.S. system, g = 32 ft/s 2.

de
Earth’s gravity always pulls downward, so the acceleration (g) of an object in free fall is always
downward and constant in magnitude, regardless of whether the object is moving up, down, or is

lh
at rest, and independent of its speed.

i
l If the object is moving downward, the downward acceleration makes it speed up; if it is moving
upward, the downward acceleration makes it slow down.

Illustration 1. Speeds of the identical cars are u and 4u at a specific instant. Find the ratio of the
respective distances of which the two cars are stopped from that instant.
v 2 - u12 u12
Solution. S1 = = ...(i)
2a 2a

v 2 - u 22 u 22 16u12
S2 = = = ...(ii)
2a 2a 2a
From (i) & (ii) S2 / S1 = 16

Illustration 2. A body travels 200 cm in first two seconds and 220 cm in next four seconds. What will
be the velocity at the end of the seventh second.
Solution Let u be the initial velocity, a be the acceleration of the body.
For first two seconds, distance travelled is 200 cm i.e., for t = 2 ; s = 200 cm.
1 2
Using second equation of motion, s = ut + at , we get,
2
u VC s v
1 s
200 = u(2) + a(2)2 or 200 = 2u + 2a
2 A C B
or u + a = 100 .....(1)
Fig.30

For next four seconds, distance travelled is 220 cm. This means for first (2 + 4) second
i.e., first 6 seconds, the distance travelled is 200 + 220 = 420 cm. Here, at t = 6 s ; s
= 420 cm. Again using second equation of motion, we get,
1
420 = u(6) + a(6)2 or 420 = 6u + 18a
2
or u + 3a = 70 .....(2)
D\Allen-Junior wing\Physics\IX\Unit-1\01-Motion\2019-20

Subtracting eq.(1) from eq.(2), we get, u + 3a = 70


– u + a = 100
––––––––––––––
2a = – 30
or a = – 15 cm/s2
Putting the value of a in eq.(1), we get, u – 15 = 100 or u = 115 cm/s
Now, we have to find velocity at the end of seventh second. Using first equation of
motion, v = u + at we get,
v = 115 + (–15)(7) = 115 – 105 = 10 cm/s

18
Ka
vi
Moti on

ra
Illustration 3. A particle moving with constant acceleration form A to B in straight line AB has

cl
velocities ‘u’ and ‘v’ at A and B respectively. Find the velocity at C, the mid point of

as
AB.

se
Solution Since C is the mid point of AB,

s
AC = CB = s (let)

de
Velocity at A, VA = u ; Velocity at B, VB = v ;

lh
Velocity at C, VC = ?

i
Applying third equation of motion between A and C, we get,
VC2 = VA2 + 2as or VC2 = u2 + 2as .....(1)
Applying third equation of motion between C and B, we get,
VB2 = VC2 + 2as or v2 = VC2 + 2asor VC2 = v2 – 2as .....(2)
Adding eq.(1) + eq.(2), we get,
VC2 + VC2 = (u2 + 2as) + (v2 – 2as)

v 2 + u2
2VC2 = v2 + u2 or VC 2 =
2

v 2 + u2
or VC =
2

Illustration 4. A particle moving with uniform acceleration in a straight line covers 3 m in the 8th
second and 5 m in the 16th second of its motion. Find the distance travelled by it from
the beginning of the 6th second to the end of the 15th second.
Solution Let u be the initial velocity, a be the acceleration of the particle. Distance covered by
the particle in 8th second is 3 m. Using the equation for snth,
1 1
3=u+ a(2 ´ 8 - 1) or 3 = u + a(15) or 2u + 15a = 6 .....(1)
2 2
Distance covered by the particle in 16th second is 5 m. Again, using the equation
for snth,

1 1
5=u+ a(2 ´ 16 - 1) or 5 = u+ a(31) or 2u + 31a = 10 .....(2)
2 2

Eq.(2) – eq.(1) Þ (2u + 31a) – (2u + 15a) = 10 – 6


or 16a = 4 or a = (1/4) m/s2

1 15 9
Using eq.(1), we get, 2u + 15 × =6 or 2u=6 - = or u = (9/8) m/s
4 4 4
D\Allen-Junior wing\Physics\IX\Unit-1\01-Motion\2019-20

Now, we have to find the distance covered by the particle from the beginning of the 6th
second to the end of the 15th second. At the beginning of the 6th second, total time
elapsed is 5 second. First, we will find the velocity at the end of 5th second using first
equation of motion,

9 æ 1ö 9 5 19
v = u + at or v= + ç ÷ (5)= + = m/s
8 è 4 ø 8 4 8

19
Ka
vi
Class IX - Physics

ra
Now time taken between the beginning of the 6th second to the end of the 15th second

cl
is actually 10 seconds (6th, 7th, 8th, 9th, 10th, 11th, 12th, 13th, 14th, 15th). [Caution

as
: If you subtract 15 – 6, you will get 9 seconds while actual time elapsed is 10 seconds]

se
s
1 2
Now, using second equation of motion, s = ut + at , we get,

de
2

lh
æ 19 ö 1 æ 1ö 190 100 290
s = ç ÷ (10) + ç ÷ (10)2 = + = = 36.25 m

i
è 8ø 2 è 4ø 8 8 8

Illustration 5. A person, on the top of a building, throws one stone vertically upwards with a velocity
'u'. He throws an another stone from the same place in the downward direction with
a velocity 'u'. Find the ratio of velocities of two stones on the bottom of the building.

Solution For the stone thrown upward, Initial velocity = + u ; acceleration, a = – g ;

distance travelled, s = – H ; final velocity = – v1.

From second equation of motion, we have, v2 = u2 + 2as

or (–v1)2 = (+u)2 + 2(–g)(–H) or v12 = u2 + 2gH or v1 = u 2 + 2gH .....(1)

For the stone thrown downward, Initial velocity = – u ; acceleration, a = – g ;

distance travelled, s = – H ; final velocity = – v2.

v2 = u2 + 2as or (–v2)2 = (–u)2 + 2(–g)(–H)

or v22 = u2 + 2gH or v2 = u 2 + 2gH .....(2)

From eq.(1) and eq.(2), we get that v1 = v2, therefore v1 : v2 = 1 : 1

D\Allen-Junior wing\Physics\IX\Unit-1\01-Motion\2019-20

20
Ka
vi
Moti on

ra
cl
as
se
Objective Questions

s
1. A particle starts from rest with a uniform acceleration. It travels a distance x in first two seconds

de
& a distance y in the next two seconds. Then :
(A) y = x (B) y = 2x (C) y = 3x (D) y = 4x

lh
i
2. A train is travelling at a speed of 90 km/h. Brakes are applied so as to produce a uniform retardation
of 0.5 m/s2. Find how far the train will go before it is brought to rest :
(A) 600 m (B) 625 m (C) 650 m (D) 675 m
–1 –2
3. A body with an intial velocity of 3 m s moves with an acceleration of 2 m s , then the distance
th
travelled in the 4 second is _______m.
(A) 10 (B) 6 (C) 7 (D) 28
–2
4. If a ball thrown vertically up attains a maximum height of 80 m, its initial speed is (g = 10 m s )
–1 –1 –1 –1
(A) 40 m s (B) 20 m s (C) 50 m s (D) 10 m s

5. A ball is released from the top of a tower of height h metres. It takes T sec. to reach the ground.
T
what is the position of the ball from ground in second :
3
h 7h 8h 17h
(A) m (B) m (C) m (D) m
9 9 9 18

Subjective Questions
6. A marble rolling on a smooth floor has an initial velocity of 0.4 m/s. If the floor offers a retardation
of 0.02 m/s2, calculate the time it will take to come to rest.

7. A car starts with velocity 10 m/s and accelerates at rate 5 m/s2. Find the final velocity when the
car has travelled a distance 30 m.
–1
8. A stone is vertically projected up with a velocity of 25 m s . Find its time of descent.
–2
(Take g = 10 m s )

9. A body is dropped from a height of 2 m. It penetrates into the sand on the ground through a
distance of 10 cm before coming to rest. What is the retardation of the body in the sand?

10. A balloon is ascending at the rate of 5 m/s at a height of 100 m above the ground when a packet
is dropped from the balloon. After how much time does it reach the ground ? (g = 10 m/s 2 )
D\Allen-Junior wing\Physics\IX\Unit-1\01-Motion\2019-20

21
Ka
vi
Class IX - Physics

ra
MOTION

cl
as
(MOT-5)

se
s
12.0 GRAPHICAL REPRESENTATION OF MOTION

de
Graph - A graph is a line, straight or curved, showing the relation between two variable quantities,

lh
of which one varies as a result of the change in the other.

i
The quantity which changes independently is called independent variable and the one which changes
as a result of the change in the other is called dependent variable.
y-a xis
12.1 Distance–Time Graph
This graph is plotted between the time taken and the distance covered.

D ista nc e (m )
The time is taken along the x-axis and the distance covered is taken
along the y-axis.
x-a xis
l The slope of the distance-time graph gives the speed of the body. T im e(s)
Fig. 31 S-t graph for a body at rest

When the body is at rest the position of the body does not change with time. The distance-time
graph of such a body is a straight line parallel to x-axis.

When the body is in uniform motion, v = constant.


600
Now, s = v × t 500
x2 B
Since v is constant 400
D ista nce

\ s µ t, thus, the distance-time graph of such a body 300 (x 2 – x 1 )


x1 200
is a straight line, inclined to x-axis. C
100

Slope = = tan q 0 2 4 6 8 10 1 2
t 1 Time t 2
x 2 - x1
Slope = t - t = v Fig. 32 S-t graph for a body in
2 1
uniform motion (a = 0)
Thus, slope of distance-time graph gives speed of the body.

l In a distance-time graph, more the slope of the graph of an object


A
in motion, more will be its speed and vice-versa.
d ista n ce

B
Example - In given graph (fig. 33) speed of particle A is greater vA > vB
than the speed of particle B because slope of graph of particle A is
greater than slope of graph of particle B.
D\Allen-Junior wing\Physics\IX\Unit-1\01-Motion\2019-20

Fig. 33 Two particles moving


with different velocities
x
l In the given graph (figure 34), speed of particle A is equal to sam e velo cit y
B
speed of particle B because graphs of both have same slope.
When the body is in non-uniform motion, distance-time graph
x0 A
is a curve. For example, in uniformly accelerated motion,
1 2
s = ut + at O t
2
Fig. 34 Two particles moving
with same velocities
22
Ka
vi
Moti on

ra
Since s µ t2, definitely the graph is not a straight line, it is a curve.

cl
as
se
distance

distan ce

s
de
lh
time t im e

i
Fig. 35 S-t graph of an accelerated Fig. 36 S-t graph of a retarded
motion (speed increasing with time) motion (speed decreasing with time)

l A distance-time graph can never be parallel to y-axis (representing distance), because this line has
inclination of 90°, and slope = tan q = tan 90° = infinite, which means infinite speed; it is impossible.
l Distance-time graph is always positive. It is always increasing, never decreasing.

12.2 Displacement-Time Graph


Here, displacement is taken on y-axis and time is taken on x-axis.
l Displacement-time graph can be positive or negative, it can be increasing or decreasing.

s
s = constant
sµt
sµt
2

s s
t t t

A body at rest A body in A body in uniformly


(s = constant) uniform motion accelerated motion
(v = 0) (s = v × t)
1
( s = ut + at 2 )
2
Fig.37 D isp lacem ent-tim e graphs for different state s of m otion

12.3 Speed-Time Graph


Here, speed is taken on y-axis and time is taken on x-axis.
l Speed-time graph is always positive, it can be increasing or decreasing.
l Area enclosed under speed-time graph gives the distance covered by the body.

v v v
v = co ns tan t
D\Allen-Junior wing\Physics\IX\Unit-1\01-Motion\2019-20

vµt

t t t
A b o dy at re st A bo dy in A b o dy in un ifo rm ly
(v = 0) u nifo rm m ot io n accelerated m o t ion
(v = consta nt) (v = u + at)

F ig .3 8 S p ee d -t im e gra p h s fo r d iffe ren t sta te s o f m o tio n

23
Ka
vi
Class IX - Physics

ra
12.4 Velocity-Time Graph

cl
The variation in velocity with time for an object moving in a straight line can be represented by

as
a velocity-time graph. In this graph, time is represented along the x-axis and velocity is represented

se
along the y-axis.
The slope of the velocity-time graph gives the acceleration of the body.

s
Area enclosed under a velocity-time graph gives the displacement of the body.

de
lh
When the body is at rest, v-t graph is a straight line lying on
the time axis. (Fig. 39)

i
Fig. 39

When the body is in uniform motion, the body moves with


constant velocity. The velocity of the body is uniform, hence the

velocity
magnitude remains same. The graph is a straight line parallel to x-
axis (time-axis). Since the velocity is uniform, its acceleration is
zero. The slope of the graph in this case is zero.

T im e
Fig. 40 v-t graph for an objelt
in uniform motion

When the body is moving with a uniform acceleration, v2

ve lo c it y
v 2– v 1
v -v v1
slope = 2 1 = tan q = a Thus, slope of v–t graph gives
t 2 - t1 t 2– t 1
acceleration of body.
O t1 t2
T im e
Fig. 41 v-t graph for an object
moving with uniform acceleration
When the body is moving with a non-uniform (variable) acceleration, velocity-time
graph is not a straight line, but is a curve.

l Speed or velocity-time graph line can never be parallel to y-axis (speed axis),
because inclination becomes 90°, then tan 90° is infinite i.e. infinite acceleration; v
it is impossible.
l Velocity-time graph can be positive or negative. It can be increasing or
t
decreasing.
Fig. 42 v-t graph for an object moving
with non-uniform acceleration
D\Allen-Junior wing\Physics\IX\Unit-1\01-Motion\2019-20

12.5 Acceleration-Time Graph


Here, acceleration is taken on y-axis and time is taken on x-axis.
l Acceleration-time graph can be positive or negative, it can be increasing or decreasing.
l Area enclosed under acceleration-time graph gives change in velocity.

24
Ka
vi
Moti on

ra
cl
a a a a = constant

as
se
t t t

s
de
lh
A bo dy at re st A b ody in A bo dy in u niform ly

i
(a = 0) uniform m otion accelerated m otion
(a = 0) (a = constant)
Fig.43 Acceleration-tim e g raphs for different state s o f m otion

13.0 EQUATIONS OF MOTION BY GRAPHICAL METHOD


Figure 44 represents a velocity-time graph, in which AB represents the initial velocity u, CE represents
final velocity v, such that the change in velocity is represented by CD, which takes place in time t,
represented by AE.

13.1 Derivation of v = u + at y-a xis


Acceleration = slope of the graph line BC C

CD CE - DE Velocity (m /s)
a= = at
BD BD B v
éQ DE = AB = u ù D
v-u êQ BD = AE = t ú u u
or a = ê ú A
t x-a xis
êë Q CE = v úû t E
tim e (second s)
v – u = at
v = u + at Fig. 44

1
13.2 Derivation of s = ut + at2
2
Distance travelled = Area of trapezium ABCE
= Area of rectangle ABDE + Area of triangle BCD
1 1
= AB × AE + (BD × CD) = u × t + [t × (v–u)]
2 2
éQ BD = AE = t ù
ê ú
ëQ CD = CE – DE = v – u û
1
=u×t + [t × (u + at – u)] [Q v = u + at]
2
1 2
s = ut + at
D\Allen-Junior wing\Physics\IX\Unit-1\01-Motion\2019-20

13.3 Derivation of v2 = u2 + 2as


From the velocity-time graph, distance covered = Area of trapezium ABCE
1 1
Þs= (AB + CE) × AE \s = (u + v) × t ...(i)
2 2
Change in velocity
Acceleration =
Time

25
Ka
vi
Class IX - Physics

ra
cl
v-u v-u
a= \t= ...(ii)

as
t a
Substituting the value of t in equation (i)

se
s
(v + u) (v - u)
s= ´ [Q A2 – B2 = (A + B) × (A – B)]
2 a

de
lh
v 2 - u2
s= , or v2 – u2 = 2as , or v 2 = u 2 + 2as

i
2a

14.0 CIRCULAR MOTION


When a particle moves along a circular path, its motion is called 'circular motion'.
v v
l In a circular motion, velocity of particle is tangential to the circular path.
If the body covers equal distances along the circumference of the circle, in
equal intervals of time, then motion is said to be a uniform circular
motion. When a body moves along a circular path, then its direction of
motion changes continuously. Thus, a circular motion is always a
non-uniform motion. v v
Fig. 45 Circular motion of a particle
l A uniform circular motion is a motion in which speed remains constant but direction of velocity
changes continuously.
Examples of uniform circular motion are (i) An athlete running on a circular track with
constant speed.
(ii) Motion of tips of the second hand, minute hand and hour hand of a wrist watch.
ACTIVE PHYSICS -Take a piece of thread and tie a small piece of stone
at one of its ends. Move the stone to describe a circular path with constant
speed by holding the thread at the other end (fig. 46). When the stone is
released, it will move in a direction tangential to the circular path. If we
release the stone from different positions direction of movement of stone will
be different for different positions. But, it is always tangential to the circular
path at the position where the stone is released. Fig. 46

14.1 Difference between Uniform Linear Motion And Uniform Circular Motion

Uniform Linear Motion Uniform Circular Motion


1 The direction of motion The direction of motion
does not change. changes continuously.
2 The motion is The motion is accelerated.
non-accelerated.
D\Allen-Junior wing\Physics\IX\Unit-1\01-Motion\2019-20

14.2 Radian B
It is the SI unit for measuring plane angles in physics.
The arc AB of the circle has length l and subtends an angle q at the centre C. l

l q A
arc C
If ÐACB = q radians Then, q = \ q= r r
radius
when l = r, then q = 1 radian.
One radian is defined as the angle subtended at the centre of the
circle by an arc which is equal in length to its radius. Fig. 47 Plane angle in
terms of radian

26
Ka
vi
Moti on

ra
Angle subtended by the circumference at the centre,

cl
2pr 360 o

as
q= = 2p radians ; 2p radians = 360° ; 1 radian =
r 2p

se
1radian = 57.3o

s
de
14.3 Angular Displacement and Angular Velocity

lh
i
Q
l
w

C r P

Fig. 48 An object in circular motion

The angle covered by a body moving along a circular path in a given interval of time is called
'angular displacement'.
The angle covered by a body per unit time is called angular velocity.
It is usually denoted by w and measured in radians per sec.
If q is the angle covered in time 't' then,
Angular displacement q
Angular velocity = \ w=
Time taken t
Unit - Angular displacement q : radian
Angular velocity w : rad/s
14.4 Relation between Linear Speed and Angular Velocity
Let us consider a particle moving along a circular path of radius 'r' with constant angular velocity
'w'. Let it covers an angle 'q' during a time interval 't'. Let the distance covered during time interval
't' be 'l'.
l
Angular displacement = q = r
l = rq ... (1) v
q
Also, angular velocity, w = ... (2)
t
l
Now, linear speed, v =
t
rq ac
or v= [using (1)]
D\Allen-Junior wing\Physics\IX\Unit-1\01-Motion\2019-20

t
æ qö
or v = rç ÷
è tø
or v = rw [using (2)] Fig. 49 Circular motion always
has radially inward acceleration
2pr
Also, linear speed =
T
where 'T' is the time period for 1 revolution.

27
Ka
vi
Class IX - Physics

ra
14.5 Centripetal Acceleration

cl
as
Circular motion is always accelerated and it has a radially inward acceleration called 'centripetal

se
acceleration'.

s
v2

de
ac =
r

lh
i
(rw)2 r 2w 2
Also, ac = = = rw 2
r r

Illustration 1. Figure shows distance-time graph of two objects A and B. Which object is moving with
greater speed when both are moving?

B
distance

time®
Fig. 50
Solution. The line for object B makes a larger angle with the time-axis. Its slope is larger than
the slope of the line for object A. Thus, the speed of B is greater than that of A.

Illustration 2. Figure 51 represents the speed-time graph for a particle. Find the distance covered by
the particle between t = 10 min. and t = 30 min.
Speed (km/h) ®

Speed (km/h) ®

15 15
D C
10 10

5 5
B
0 10 20 30 40 10 20 30 40
time (min) ® time (min) ®
(a) (b)
Fig. 51
Solution. We draw perpendicular lines from the 10-minute point and the 30-minute point to the
D\Allen-Junior wing\Physics\IX\Unit-1\01-Motion\2019-20

line of graph (see fig. 51 (b)). The distance covered is equal to the area of the rectangle
ABCD. Its value is
ABCD = (30 min.–10 min.) × (10 km/h)
= 20 min. × 10 km/h

20 10
= h × 10 km/h = km.
60 3

28
Ka
vi
Moti on

ra
Illustration 3. Find the distance coverd by a particle during the time interval t = 0 to t = 20 s for

cl
which the speed-time graph is shown in figure.

as
se
s
20

Speed (in m/s) ®

de
15

lh
i
10

0 5 10 15 20 25
time (in sec.) ®
Fig. 52
Solution. The distance covered in the time interval 0 to 20 s. is equal to the area of the shaded
triangle. It is
1 1
× base × height = × (20 s) × (20 m/s) = 200 m.
2 2

Illustration 4. What is the angular velocity in rad/s of the hour, minute and second hand of clock?
Solution Time period of revolution of hour hand, T 1 = 12 hours = 12 × 60 × 60 s
2p 2p p
Angular velocity of hour hand, w1 = = = rad/s
T1 24 ´ 60 ´ 60 43200
Time period of revolution of minute hand, T2 = 1 hour = 1 × 60 × 60 s
2p 2p p
Angular velocity of minute hand, w2 = = = rad/s
T2 1´ 60 ´ 60 1800
Time period of revolution of second hand, T 3 = 1 minute = 1 × 60 s
2p 2p p
Angular velocity of second hand, w3 = = = rad/s
T3 1´ 60 30

Illustration 5. A particle is moving with constant speed in a circular path. Find the ratio of average
p
velocity to its instantaneous velocity when the particle describes an angle q = .
2
q qR pR
Solution Time taken to describe angle q, t = = =
w v 2v
Total displacement 2R 2 2
Average velocity = = = v
totaltime p R/2v p
Instantaneous velocity = v
2 2
The ratio of average velocity to its instantaneous velocity = ans.
p
D\Allen-Junior wing\Physics\IX\Unit-1\01-Motion\2019-20

29
Ka
vi
Class IX - Physics

ra
cl
as
Objective Questions

se
1. Motion represented in the following graph is :

s
(A) uniform

de
(B) Accelerated x
(C) Retarded

lh
(D) None of these

i
t

2. The variation of velocity of a particle moving along a straight line is shown


in the figure. The distance travelled by the particle in 4s is: 20
(A) 25m (B) 30m
(C) 55m (D) 60m 10

1 2 3 4
3. A cyclist goes around a circular track once every 2 minutes. If the radius of the circular track is 105
m, what will be his speed? (Take p = 22/7)
(A) 2.5 m/s (B) 3.5 m/s (C) 4.5 m/s (D) 5.5 m/s

4. Which of the following statement is not true?


(A) The motion along circular path is accelerated
(B) A force is needed to produce a circular motion
(C) The direction of velocity along circular motion remains unchanged
(D) A body moving along circular path may have uniform speed

5. The velocity- time graph of a body moving in a straight line is v(m/sec.)


shown in figure. The displacement and distance travelled by 5
the body in 6 seconds are respectively.
4
(A) 8 m, 16 m 3
(B) 16 m, 8 m 2
1
(C) 16 m, 16 m 0 (sec.)
1 2 3 4 5 6 t
–1
(D) 8 m, 8 m
–2
Subjective Questions
6. What is represented by the slope of v–t graph ?

7. Figure shows the velocity-time graphs for two objects, A and B,


moving along the same direction. Which object has greater
acceleration ?

8. State with reasons which of these can not possibly represent one-dimensional motion of a particle.
D\Allen-Junior wing\Physics\IX\Unit-1\01-Motion\2019-20

9. A body covers a semicircle of radius 7 cm in 5 sec. Find its speed.

10. Uniform circular motion is an example of accelerated motion. Explain.

30
Ka
vi
Moti on

ra
cl
as
se
l Electric current has a direction, still it is considered as a scalar quantity. It does not follow the vector

s
laws of addition, it simply follows the scalar (algebraic) laws.

de
l Two vectors are equal if they have the same magnitude and the same direction.

lh
l Arrows (or rays) are used to represent vectors. The direction of the arrow gives the direction of the

i
vector. The length of the arrow is proportional to the magnitude of the vector.

P Q

l Distance ³|Displacement|

Distance is equal to magnitude of displacement when a body moves in a straight line in a particular
direction or it is in uniform motion.

l The displacement of a particle in a given interval of time is independent of choice of origin.

l If an object moves without rotation, its motion is called translational motion. A translational motion
can have straight or curved path.

l In uniform motion, velocity of particle remains constant i.e., its magnitude as well as direction are
constant.

l In uniform motion, average speed/velocity is equal to instantaneous speed/velocity at any point of


time.

Speed ³ |Velocity|.

l Speed is equal to velocity when a particle moves in a straight line without change in direction.

l Direction of velocity represents direction of motion of body. Also, sign of velocity represents the
direction of motion of body.

l If acceleration of a particle is zero, this means its velocity is constant i.e. the particle is in uniform
motion.

l In uniformly accelerated motion, acceleration is constant in both magnitude as well as direction.

l The acceleration needed to keep an object moving in a circular path is called centripetal acceleration.
It always acts towards the centre of circular path.
D\Allen-Junior wing\Physics\IX\Unit-1\01-Motion\2019-20

31
Ka
vi
Class IX - Physics

ra
cl
as
1. A body covers a distance of 5 m along a semicircular path from its one end to another end. Then

se
the ratio of its distance covered to its displacement is :

s
(A) 11 : 7 (B) 12 : 5 (C) 8 : 3 (D) 7 : 5

de
lh
2. A dog runs 120 m away from its master in a straight line in 9.0s, and then runs halfway back in
one-third the time. Calculate its average speed

i
(A) 10m/s (B) 15m/s (C) 20m/s (D) 5m/s

3. A particle covers each 1/3 of the total distance with speed V1, V2 and V3 respectively. Find the
average speed of the particle:
V1 V2 V3 2V1V2 V3 3V1V2 3V1V2V3
(A) V V + V V + V V (B) V V + V V + V V (C) V V + V V (D) V V + V V + V V
1 2 2 3 1 3 1 2 2 3 1 3 1 2 2 3 1 2 2 3 1 3

4. A farmer has to go 500 m due north, 400 m due east and 200 m due south to reach his field.
If he takes 20 min to reach the field, what is the average velocity of farmer during the walk ?
(A) 35 m/min. (B) 45 m/min. (C) 25 m/min. (D) 55 m/min.

5. The position of a particle going along a straight line is x1=100m at 10:00 A.M. and x2=200m at
10:30 P.M. The velocity of particle is:
10:00 AM 10:30 PM

Origin X
A B
100 m
200 m
(A) 0.3m/min (B) 0.133m/min (C) 6.66m/min (D) 0.5m/min

6. Choose the correct statement:


(A) A body having zero velocity will not necessarily have zero acceleration.
(B) A body having zero velocity will necessarily have zero acceleration.
(C) A body having uniform speed has uniform acceleration.
(D) A body having non-uniform velocity has zero acceleration.

7. Two trains of length 500 m and 1000 m moving in opposite direction with same speed crosses
each other in 10 sec, find their speed ?
(A) 75 m/s (B) 150 m/s (C) 100 m/s (D) None of these

8. Two boys start running towards each other from two points, they are 120 m apart. One runs with
D\Allen-Junior wing\Physics\IX\Unit-1\01-Motion\2019-20

a speed of 5 m/s and other with a speed of 7 m/s. When and where do they meet each other from
first point ?
(A) 10 s, 50 m (B) 10 s, 70 m (C) 24 s, 50 m (D) 17 s, 70 m

9. If the time of fall of two objects are in the ratio 1 : 2, find the ratio of the heights from which they
fall?
(A) 1: 2 (B) 2: 1 (C) 1: 4 (D) 4: 1

32
Ka
vi
Moti on

ra
10. A stone dropped from top of a tower reaches the ground in 4s. The height of the tower is about:

cl
(Take g = 10ms–2)

as
(A) 20m (B) 40m (C) 80m (D) 160m

se
s
11. A body falls freely from rest. It covers as much distance in the last second of its motion as covered

de
in first three seconds. The body has fallen for a time of:
(A) 3s (B) 5s (C) 7s (D) 9s

lh
i
12. A ball dropped from a height covers half of its total journey from top of a tower in 0.5s. The height
of the tower is: (Take g = 9.8ms–2)
(A) 4.9 m (B) 9.8 m (C) 2.45 m (D) 8.6 m

13. A car moving with an initial velocity u is brought to rest by application of brakes which provides
uniform retardation of 2.5ms–2 for 10s. The value of u is:
(A) 20ms–1 (B) 25ms–1 (C) 30ms–1 (D) 35ms–1

14. A stone is thrown vertically upward with an initial velocity u from the top of a tower. It reaches
the ground with a velocity 3u. The height of the tower is

3u 2 4u 2 6u 2 9u 2
(A) (B) (C) (D)
g g g g

15. A person standing near the edge of the top of a building throws two balls A and B. The ball A is
thrown vertically downward and the ball B is thrown vertically upward with the same speed. The
ball A hits the ground with a speed vA and the ball B hits the ground with a speed vB . We have.
(A) vA > vB
(B) vA < vB
(C)vA = vB
(D) The relation between A and B depends on height of the building above the ground.

16. The distance of a particle as a function of time is shown below. The ¥


graph indicates that d
(A) The particle starts with certain velocity but the motion is retarded
and finally the particle stops
(B) The velocity of the particle is constant throughout
(C) The acceleration of the particle is constant throughout in the 0 t
direction of motion
(D) The particle starts with some constant velocity, the motion is accelerated, and finally the
particle moves with some constant velocity.
D\Allen-Junior wing\Physics\IX\Unit-1\01-Motion\2019-20

17. Figure BC represents a body moving :


Displacement

(A) backward with uniform velocity


(B) forward with uniform velocity A B
(C) backward with non uniform velocity
(D) forward with non uniform velocity O time
C

33
Ka
vi
Class IX - Physics

ra
18. Which of the following distance–time graphs is not possible :

cl
as
Distance Distance

se
s
de
(A) (B)

lh
Time Time

i
Distance Distance

(C) (D)

Time Time

19. The speed of a particle in distance–time graph given below is :


Time
5 sec

0 20 m Distance

(A) 1/4 m/sec (B) 4 m/sec (C) 10 m/sec (D) zero

20. In the graph given below :


(A) The body first accelerates & then decelerates
x
(B) The body first decelerates & then accelerates
(C) Body has negative velocity
(D) None of these t

21. Two persons A & B are moving along a straight line & their motion is represented in the graph
shown. Who has a greater velocity :

B
A
D\Allen-Junior wing\Physics\IX\Unit-1\01-Motion\2019-20

t
(A) A (B) B (C) Cannot be predicted (D) both have equal velocity

34
Ka
vi
Moti on

ra
22. The area shown in Figure represents :

cl
(A) momentum

as
se
(B) acceleration

Velocity

s
(C) displacement

de
lh
(D) speed

i
Time
23. For the velocity time graph given below :

V
10 m/s
15s 20s
0
5s 10s t(s)
–10 m/s

(A) Displacement in 20s is 75 m (B) Distance travelled in 20s is 125 m


(C) The body has taken only one turn (D) All the above

24. The velocity versus time graph of a body moving along a straight line is as shown in fig. The ratio
of displacement and distance covered by body in 5 seconds is:
v (m/s)
2

0
1 2 3 4 5 t (s)
–1

–2

(A) 2 : 3 (B) 3 : 5
(C) 1 : 1 (D) 1.5 : 5

25. If a body is moving with a constant speed of 10ms–1 in a circular path of radius 2m. The angular
velocity of the body is:
D\Allen-Junior wing\Physics\IX\Unit-1\01-Motion\2019-20

(A) 10 rad/s (B) 5 rad/s (C) 7.5 rad/s (D) 2 rad/s

35
Ka
vi
Class IX - Physics

ra
cl
SEASONED

as
1. A particle moving in a straight line covers half the distance with speed 3 m/s and the other half of

se
the distance is covered in two equal time intervals with speed of 4.5 m/s and 7.5 m/s respectively.

s
Then the average speed of particle during this motion is :

de
(A) 4m/s (B) 4.5 m/s (C) 5 m/s (D) 5.5 m/s

lh
2. Particle has initial velocity 9 ms–1 due east and constant acceleration of 2 ms–2 due west. If the

i
n
distance covered by it in fifth second of its motion is m , then the value of ‘n’ is :
10
(A) 5 (B) 10 (C) 15 (D) 20
3. A car starts from rest and moves with constant acceleration. Then the ratio of the distance covered
by it in the nth second to that covered in n seconds is :
2 1 2 1 2 1 2 1
(A) 2
- (B) + (C) - (D) +
n n n2 n n n2 n n2

4. A body A starts from rest with an acceleration a1. After 2s, another body B starts from rest with an
acceleration a2. If they travel equal distances in 5th second after start of motion of A, the ratio of
a1:a2 is:
(A) 5:9 (B) 9:5 (C) 5:7 (D) 7:5
a (ms–2)
5. The acceleration of a cart started at t = 0, varies with time
as shown in figure. The car starts from rest. Then the 5
distance travelled by cart in 30 seconds is:
20 30 t (s)
(A) 100m (B) 1000 m 0
10
(C) 1500 m (D) 3000 m
–5

6. A driver driving a truck at a constant speed of 20 ms–1 suddenly saw a parked car ahead of him by
95 m. He could apply the brake after some time to produce retardation of 2.5 ms–2. An accident
was just avoided, his reaction time is :
(A) 0.5 s (B) 0.75 s (C) 0.8 s (D) 1 s
7. A body is released from a great height and falls freely towards the earth. Another body is released
from the same height one second later. Then, the separation between the two bodies, two seconds
after release of the second body is : (Take g = 9.8 ms–2)
(A) 9.8 m (B) 4.9 m (C) 24.5 m (D) 19.6 m
8. Between two stations a train accelerates uniformly at first, then moves with constant speed and
finally retards uniformly. If the ratios of time taken are 1 : 8 : 1 and the greatest speed is 60 km/hr,
find the average speed over the whole journey.
(A) 60 km/hr (B) 54 km/hr (C) 30 km/hr (D) 20 km/hr
D\Allen-Junior wing\Physics\IX\Unit-1\01-Motion\2019-20

9. A hot air balloon starts rising from ground with an acceleration of 1.25ms–2. After 8s, a stone is
dropped from it. The stone will:
(A) cover a distance of 40m before hitting the ground
(B) have a displacement of 50m on hitting the ground
(C) reach the ground in 4s
(D) start moving downward instantly after being released
10. A bus is moving with a constant velocity 10 m/s on a straight road. A scooterist wishes to overtake
the bus in 100 s. If the bus is at a distance of 1 km from the scooterist, then the velocity with which
scooterist should chase the bus is :
(A) 50 m/s (B) 40 m/s (C) 30 m/s (D) 20 m/s
36
Ka
vi
Moti on

ra
cl
CBSE PATTERN

as
1
1.

se
A particle is moving in a circle of diameter 5 m. What is its displacement when it complete 1
2

s
revolutions.

de
2. A body thrown in the vertically upward direction rises upto a height h and comes back to the

lh
position of start. Calculate:

i
(A) The total distance travelled by the body.
(B) The displacement of the body.

3. An object starts with initial velocity u and attains a final velocity of u. The velocity of the ojbect is
changing at a uniform rate. Write the formula for calculating the average velocity Vau.

4. What kind of motion of a body is represented by the graphs given below?


Distance

Distance

Time Time
(a) (b)

5. Why is the motion of an athelte moving along the circular path an accelerated motion?

6. Four speed-time graphs are shown below:


y y y y
Speed(m/s)

Speed(m/s)

Speed(m/s)

Speed(m/s)

x x x
Time(s) Time(s) Time(s) Time(s)
(a) (b) (c) (d)

Which graph represents the following case?


(a) A ball thrown vertically upwards and returning to the band of the thrower?
(b) A body decelerating to a constant speed and then accelerating.
Speed (m/s) Time(s)
(a) (b)
(c) (d)
D\Allen-Junior wing\Physics\IX\Unit-1\01-Motion\2019-20

7. What do the graphs shown below indicate?


y y
Speed (ms–1)

Speed (ms–1)

x x
Time (s) Time (s)

37
Ka
vi
Class IX - Physics

ra
8. The graph given below is the distance-time graph of an object. y

cl
100

as
(i) Find the speed of the object during first four seconds of 75

Distance(m)
50 P Q

se
its journery.
25

s
(ii) How long was it stationary? 25
R

de
x
0 2 4 6 8 10 12 14 16
(iii) Does it represents a real situation? Justify your answer.

lh
Time(s)

i
9. Joseph jogs from one end A to the other end B of a straight 300 m road in 2 minutes 50 seconds
and then turns around and jogs 100 m back of point C in another 1 minute. What are Joseph's
average speeds and velocities in jogging?

10. Rajeev went from Delhi to Chandigarh on his motorbike. The odometer of the bike read 4200 km
at the start of trip and 4460 km at the end of his trip. If Rajeev took 4h 20 minutes to complete his
tirp, find the average speed in kmh–1 as well as in ms –1.

11. Starting from a stationary position, Rehan paddles his bicycle to attain a velocity of 6 m/s in 30 s.
Then he applies brakes such that the velocity of the bicycle comes down to 4 m/s in the next 5 s.
Calculate the acceleration of the bicycle in the cases.

12. A car is moving on straight road with a uniform acceleration. The following table gives the speed
of the car at various instants of time:

Time (s) 0 10 20 30 40 50

Speed(ms–1) 5 10 15 20 25 30

(i) Draw the shape of speed-time graph representing the above sets of observations.

(ii) Find the acceleration of the car. [CBSE 2010]

13. The V-T graph of cars A and B which starts from the same
place and move along a straight road in the same direction, is
A

90
ar

shown.
C

80 A
70
Calculate : 60
D E CarB
50
V(m/s)

(i) The acceleration of car A between 0 and 8 s. 40


30
(ii) The acceleration of car B between 2 s and 4 s. 20
D\Allen-Junior wing\Physics\IX\Unit-1\01-Motion\2019-20

10
(iii) The points of time at which both the cars have the same C B
0 1 2 3 4 5 6 7 8 9
velocity.
Time(s)
(iv) Which of the two cars is ahead after 8 sec. and by how much? [CBSE 2010]

38
Ka
vi
Moti on

ra
14. The distance-time graph of two trains are given below. The trains start simultaneously in the same

cl
direction.

as
Y

se
A
200
B

s
Q

de
150

lh
P

Distance(km)
100 R

i
50

S
X
0 1 2 3
Time(h)
(i) How much ahead of A is B when the motion starts?
(ii) What is the speed of B?
(iii) When and where will A catch B? [CBSE 2010]

15. An object starts linear motion with a velocity 'u' and under uniform acceleration a it acquires a
velocity 'u' in time 't'. Draw velocity-time graph. From this graph obtain the following equations.
(a) u = u + at
1
(b) S = ut + at2 [CBSE 2010]
2
D\Allen-Junior wing\Physics\IX\Unit-1\01-Motion\2019-20

39
Ka
vi
Class IX - Physics

ra
cl
as
1. A student starts with a velocity 40 km/hr for school at 4 km away from his house. Due to closing of

se
school he returns soon to his house with a velocity of 60 km/hr. His average velocity will be :

s
(Rajasthan/NTSE Stage - I 2007)

de
(A) zero (B) 10 km/hr (C) 48 km/hr (D) 50 km/hr

lh
2. A graph given, shows the variation of velocity and time of two bodies A and B. Choose an alternative

i
for their average velocities (NSEJS 2009)

(A) Average velocities of both are same since they have same initial and final velocities
(B) Average velocities of both are same since both cover equal distance in equal interval of time
(C) Average velocity of A is greater than that of B since it covers more distance than B in 10 sec.
(D) Nothing can be said since their accelerations are not given

3. A ball is thrown vertically upwards. Ignore air resistance. Take the upward motion as positive.
Which one of the following graphs represents the velocity of the ball as a function of time? (Time is
plotted along the horizontal axis in all cases.) (NSEJS 2009)

(A) (B) (C) (D)

4. Which of the given velocity - time graphs matches the given acceleration- time graph which you see
at the right? (Time is plotted along the horizontal axis in all cases.) (NSEJS 2009)

t
D\Allen-Junior wing\Physics\IX\Unit-1\01-Motion\2019-20

(A) (B) (C) (C)

40
Ka
vi
Moti on

ra
5. There are two tracks A and B as shown in the figure. The direction of gravity is also shown in the

cl
figure. If two similar balls begin to move at same uniform velocity at the same time which of the two

as
balls will reach the end of the track first? (INJSO 2010)

se
s
g

de
lh
i
(A) Ball on track A (B) Ball on track B
(C) They will reach on the same time (D) Cannot decide by the data given.

6. Let there be a rigid wheel rolling without sliding on a horizontal surface.

The path of point ‘A’ as seen by an observer on the ground, when the wheel is moving along x
axis is: (INJSO 2010)

(A) (B)

x x

(C) (D)
x
x

7. A velocity - time graph for a moving object is shown below. What would be the total displacement
during time t = 0 to t = 6s ? (Orissa/NTSE Stage - I 2013)
D\Allen-Junior wing\Physics\IX\Unit-1\01-Motion\2019-20

5
v
6 t(s)
m/s
0
1 2 3 4 5 7 8 9 10

–5

(A) 10 m (B) 20 m (C) 2.5 m (D) 0.0 m

41
Ka
vi
Class IX - Physics

ra
8. A ball hits a wall horizontally with a velocity of 6.0 ms–1. After hitting wall it rebounds horizontally

cl
with a velocity of 4.4 ms–1. If the balls remains in the contact of wall for 0.040 sec. The acceleration

as
of ball would be (take direction of initial velocity as positive)

se
(Uttarakhand/NTSE Stage - I 2014)

s
(A) –260 m/s2 (B) +260 m/s2 (C) –26 m/s2 (D) +26 m/s2

de
9. A body covers half the distance with a speed of 20 m/s and the other half with 30 m/s. The average

lh
speed of the body during the whole journey is : (West Bengal/NTSE Stage - I 2014)

i
(A) Zero (B) 24 m/s (C) 25 m/s (D) None of the above

10. A body starts from rest at time t = 0, the acceleration time graph is shown in figure. The maximum
velocity attained by the body will be : (Delhi/NTSE Stage - I 2014)

10

2
a(m/s )

O 11 t(s)

(A) 1110 m/s (B) 55 m/s (C) 650 m/s (D) 550 m/s

11. A body falling from rest describes distances S1, S2 and S3 is the first, second and third seconds of its
fall. Then the ratio of S1 : S2 : S3 is (Delhi/NTSE Stage - I 2014)
(A) 1 : 1 : 1 (B) 1 : 3 : 5 (C) 1 : 2 : 3 (D) 1 : 4 : 9

12. Value of one Fermi is : (Madhya Pradesh/NTSE Stage - I 2014)


(A) 10–13 metre (B) 10–14 metre (C) 10–15 metre (D) 10–16 metre

13. The graph below describe the motion of a ball rebounding from a horizontal surface being released
from a point above the surface. (Haryana/NTSE Stage - I 2014)

t
The quantity represented in the y-axis is the ball's :
(A) Displacement (B) Velocity (C) Acceleration (D) Momentum

14. The acceleration versus time graph of an object is as shown in figure. The corresponding velocity-
time graph of the objects is : (Rajasthan/NTSE Stage - I 2014)
D\Allen-Junior wing\Physics\IX\Unit-1\01-Motion\2019-20

v v v v
(A) (B) (C) (D)
t t t t
42
Ka
vi
Moti on

ra
15. A bullet of mass 10g travelling horizontally with a velocity of 160 ms –1 strikes a stationary wooden

cl
block and comes to rest in 0.02 s. The distance of penetration of the bullet into the block will be:

as
(Rajasthan/NTSE Stage - I 2014)

se
(A) 1.20 m (B) 1.60 m (C) 2.00 m (D) 2.40 m

s
16. The velocity-time graph of a body falling from rest under gravity and rebounding from a solid

de
surface is represented by (Rajasthan/NTSE Stage - I 2014)

lh
i
v
v v v

(A) (B) (C) (D)


t
t t O t
O O O

17. A ball is thrown vertically upwards with a given velocity 'u' such that it rises for T seconds (T > 1),
What is the distance traversed by the ball during the last one second of ascent (in meters) ?
(Acceleration due to gravity is g m/s2.) (NTSE Stage - II 2015)
1 2 1
(A) gT (B) vT + g[T 2 - (T - 12 )]
2 2
g 1
(C) (D) g[T 2 - (T - 1)2 ]
2 2

18. A man running with a uniform speed 'u' on a straight road observes a stationary bus at a distance
'd' ahead of him. At that instant, the bus starts with an acceleration 'a'. The condition that he would
be able to catch the bus is- (NTSE Stage - II 2015)

u2 u2 u2 u2
(A) d £ (B) d £ (C) d £ (D) d £
a 2a 3a 4a

19. A stone is dropped from the top of a tower. Its velocity after it has fallen 20 m is (take g = 10 m/s2)
(Bihar/NTSE Stage - I 2015)
(A) –10 m/s (B) 10 m/s (C) 30 m/s (D) 20 m/s

20. A body is dropped from certain height from a uniformly ascending balloon. The correct graph
showing variation of velocity with time for body is : (Haryana/NTSE Stage - I 2015)

(A) (B)
D\Allen-Junior wing\Physics\IX\Unit-1\01-Motion\2019-20

(C) (D)

43
Ka
vi
Class IX - Physics

ra
21. Which motion does the graph of distance and time shows for accelerated motion ?

cl
(Gujarat/NTSE Stage - I 2015)

as
(A) Non uniformly accelerated

se
Distance

s
(B) Constant velocity

de
(C) Uniformly accelerated

lh
(D) Uniformly retarded motion Time

i
22. A car travels 40 kms at an average speed of 80 km/h and then travels 40 kms at an average speed
of 40 km/h. The average speed of the car for this 80 km trip is: (Rajasthan/NTSE Stage - I 2015)
(A) 40 km/h (B) 45 km/h (C) 48 km/h (D) 53 km/h

23. A particle starts its motion from rest under the action of a constant force. If the distance covered in
first 10s is S1 and that covered in first 20s is S2 then. (Delhi/NTSE Stage - I 2015)
(A) S2 = S1 (B) S2 = 2S1 (C) S2 = 3S1 (D) S2 = 4S1

24. A ball thrown vertically upward returns to the thrower after 6s. The ball is 5 m below the highest
point at t = 2s. The time at which the body will be at same position, (take g = 10 m/s2)
(Delhi/NTSE Stage - I 2015)
(A) 2.5s (B) 3s (C) 4s (D) 5s

25. The speed of a train decreases from 80 km/hour to 60 km/hour in 5 seconds. In the process, find out
the acceleration of the train. (Uttarakhand/NTSE Stage - I 2015)
2 2
(A) 2.22 m/sec (B) –2.22 m/sec (C) –1.11 m/sec2 (D) 1.11 m/sec2

26. Correct relation is ............. (Madhya Pradesh/NTSE Stage - I 2015)


(A) v2 = u2 + 2a2s2 (B) v2 = u2 – 2a2s2
(C) v2 = u2 + 2as (D) v2 = u2 + 2a2s

27. A person takes time t to go once around a circular path of diameter 2R. The speed (v) of this person
would be: (Rajasthan/NTSE Stage - I 2015)

t 2pR pR 2
(A) (B) (C) (D) 2pR.t
2pR t t

28. A body thrown vetically up reaches a maximum height and returns back. Its acceleration is–
(NSEJS-2016-17)
(A) Downward during both ascent and descent
D\Allen-Junior wing\Physics\IX\Unit-1\01-Motion\2019-20

(B) Downward at all positions except at the highest point, where it is zero
(C) Upward during both ascent and descent
(D) Downward during ascent and upward during descent

29. The brakes applied to a car produce an acceleration of 8 m/s2 in the opposite direction to the
motion. If the car takes 3 seconds to stop after the application of brakes, the distance it travels
during the time will be– (Rajasthan/NTSE Stage - I 2016)
(A) 30 m (B) 36 m (C) 25 m (D) 40 m

44
Ka
vi
Moti on

ra
30. Consider the following five graphs (note the axes carefully). Which of the following represents motion

cl
at constant speed ? (NTSE Stage - I 2017)

as
se
s
acceleration
Distance

Velocity
Speed

de
(A) (B) (C) (D)

lh
Time Time Time Time

i
Acceleration

(E)

Time
(A) D only (B) D and E (C) A, B and C (D) A and D

31. A ball is shot vertically upward with a given initial velocity. It reaches a maximum height of 100 m.
If on a second shot, the initial velocity is doubled then the ball will reach a maximum height of
(NTSE Stage - I 2017)
(A) 70.7 m (B) 141.4 m (C) 200 m (D) 400 m.

32. The position of two blocks at successive 0.20-second time intervals are represented by the numbered
squares in the figure below. The blocks are moving towards right.
(NSEJS-2017-18)

Block a 1 2 3 4 5 6 7

Block b
1 2 3 4 5

The acceleration of the blocks are related as follows :


(A) acceleration of ‘a’ is greater than acceleration of ‘b’.
(B) acceleration of ‘a’ equals acceleration of ‘b’. Both accelerations are greater than zero.
(C) acceleration of ‘b’ is greater than acceleration of ‘a’
(D) acceleration of ‘a’ equals acceleration of ‘b’. Both acceleration are zero

33. If x, v and t represent displacement (m), velocity (m/s) and time (s) respectively for a certain particle
then which pair of the following graphs can be best correlated to each other.
(NSEJS-2017-18)
D\Allen-Junior wing\Physics\IX\Unit-1\01-Motion\2019-20

x v v v

t t t t
Fig.I Fig.II Fig.III Fig.IV
(A) I & II (B) I & III (C) I & IV (D) None of these

45
Ka
vi
Class IX - Physics

ra
34. The velocity-time graph of motion of two cars A and B is shown in the figure

cl
(NTSE Stage - II 2018)

as
Car B

se
s
Car A

de
Velocity

lh
(v)

i
t0
time (t) ¾®

Choose the correct statement.


(A) Accelerations of two cars are equal to each other at time t = t0.
(B) Accelerations of two cars are equal to each other at an instant greater than t0.
(C) Accelerations of two cars are equal to each other at an instant earlier than t0.
(D) At no instant in the interval 0 £ t £ t0, the two accelerations are equal.

35. Velocity-time graph of a body moving with uniform acceleration is shown in the diagram. The
distance travelled by the body in 3 seconds is (NTSE Stage - I 2018)

30
velocity(m/s)

20

10

0 Time(s)
1 2 3
(A) 90 m (B) 45 m (C) zero (D) 10 m

36. A tiny ball of mass m is initially at rest at height H above a cake of uniform thickness h. At some
moment the particle falls freely, touches the cake surface and then penetrates in it at such a constant
rate that its speed becomes zero on just reaching the ground (bottom of the cake). Speed of the ball
at the instant it touches the cake surface and its retardation inside the cake are respectively
(NSEJS-2018-19)

æH ö æH ö
(A) 2gh and g ç - 1÷ (B) 2g(H - h) and g ç - 1÷
èh ø èh ø

æh ö æh ö
(C) 2gh and g ç - 1÷ (D) 2g(H - h) and g ç - 1÷
èH ø èH ø

37. Two particles P1 and P2 move towards origin O, along X and Y-axes (NSEJS-2018-19)
D\Allen-Junior wing\Physics\IX\Unit-1\01-Motion\2019-20

at constant speed u1 and u2 respectively as shown in the figure. At t O


P1
= 0, the particles P1 and P2 are at distances a and b respectively u1
from O. Then the instantaneous distance s between the two particles
u2
is given by the relation.
(A) s = [a2 + b2 + (u12 + u22)t2 – 2t(au1 + bu2)]1/2
(B) s = [a2 + b2 + (u12 + u22)t2 – 2t(bu1 + au2)]1/2 P2
(C) s = [a + b + (u + u )t + 2t(au1 + bu2)]
2 2
1
2
2
2 2 1/2

(D) s = [a2 – b2 + (u12 + u22)t2 – 2t(au1 + bu2)]1/2

46
Ka
vi
Moti on

ra
ANSWERS

cl
as
CHECK POST-1

se
1. (A) 2. (B) 3. (A) 4. (D) 5. (B)

s
de
CHECK POST-2

lh
1. (A) 2. (B) 3. (A) 4. (C) 5. (A) 6. Yes

i
7. Distance = 2200 m, Displacement = 200 m 8. (i) 800 m (ii) 0

9. Distance = 235.7 m, Displacement = 50 2 m 10. Distance=140m, Displacement = 100 m

CHECK POST-3
1. (C) 2. (B) 3. (C) 4. (B) 5. (A)

CHECK POST-4
1. (C) 2. (B) 3. (A) 4. (A) 5. (C) 6. 20 s
2
7. 20m/s 8. 2.5 s 9. 200 m/s 10. 5 s

CHECK POST-5
1. (B) 2. (C) 3. (D) 4. (C) 5. (A)

EXERCISE-1 : (ELEMENTARY)
Que. 1 2 3 4 5 6 7 8 9 10
Ans. A B D C B A A A C C
Que. 11 12 13 14 15 16 17 18 19 20
Ans. B C B B C A A D B B
Que. 21 22 23 24 25
Ans. B C D B B

EXERCISE-2 : (SEASONED)
Que. 1 2 3 4 5 6 7 8 9 10
Ans. A A C A B B C B C D

EXERCISE-3 : (CBSE PATTERN)


D\Allen-Junior wing\Physics\IX\Unit-1\01-Motion\2019-20

u+ v
1. 5 m (from initial position to final position) 2. 2h, 0 3. v avg =
2
8. (i) 12.5 m/s 9. 1.74 m/s, 0.87 m/s 10. 60 km/h, 16.67 m/s
11. 0.2 ms-2,–0.4m/s2 12. (ii) 0.5 ms-2
13. (i) 10ms-2 (ii) 20ms-2 (iii) at t = 2s and at t = 6s (iv) B is ahead of A by 10 m
14. (i) 100 km (ii) 25 km/h
(iii) After 2 hours A will catch B at a distance of 150 km from the starting point of A.

47
Ka
vi
Class IX - Physics

ra
cl
EXERCISE-4 : (COMPETITIVE ASSESSMENT)

as
Que. 1 2 3 4 5 6 7 8 9 10

se
Ans. A C C A B A A A B B

s
Que. 11 12 13 14 15 16 17 18 19 20

de
Ans. B C A D B C C B D A

lh
Que. 21 22 23 24 25 26 27 28 29 30

i
Ans. C D D C C C B A B D
Que. 31 32 33 34 35 36 37
Ans. D D B C B B A

D\Allen-Junior wing\Physics\IX\Unit-1\01-Motion\2019-20

48
Ka
vi
ra
cl
as
se
s
Chapter

de
2

lh
i
Force and Laws of Motion
1.0 INTRODUCTION
2.0 EFFECTS OF FORCE
3.0 BALANCED AND UNBALANCED FORCE
3.1 Balanced Force
3.2 Unbalanced Force
4.0 GALILEO'S INCLINED PLANES
5.0 NEWTON'S FIRST LAW OF MOTION
5.1 Concept of Inertia
5.2 Inertia of Rest
5.3 Inertia of Motion
5.4 Inertia of direction
6.0 LINEAR MOMENTUM (OR MOMENTUM)
7.0 NEWTON'S SECOND LAW OF MOTION
7.1 Applications / Results of Newton's Second Law
8.0 IMPULSE OF A FORCE
9.0 NEWTON'S THIRD LAW OF MOTION
9.1 Examples / Applications of Newton's Third Law
10.0 CONSERVATION OF MOMENTUM
Unit One

11.0 THE NORMAL FORCE AND TENSION IN THE STRING


11.1 Normal Force
11.2 Tension in the String
EXERCISE-1 (ELEMENTARY)
EXERCISE-2 (SEASONED)
EXERCISE-3 (CBSE PATTERN)
EXERCISE-4 (COMPETITIVE ASSESSMENT)
i
lh
de
s
se
as
cl
ra
vi
Ka
LL
Ka
vi
ra
Force and Laws of Motion

FORCE AND LAWS OF MOTION (NLM)

cl
as
(NLM-1)

se
s
1.0 INTRODUCTION

de
In common usage, a force is a push or a pull on some object. If you pull hard enough on a trolley

lh
[see fig. 1(a)], the trolley moves. When you kick a football [see fig.1(b)], it deforms slightly and is set

i
in motion. If you pull on a spring [see fig.1(c)], the spring stretches. If you pull a drawer [see
fig.1(d)], it will move out of the rack. Forces such as those that pull the spring or launch the football
are called contact forces, because they arise from the physical contact between two objects.

(a) A trolley moves when it is pulled (b) A foot ball moves when it is kicked

Force

Force

(c) A spring stretches when it is pulled (d) The drawer moves out when it is pulled
Fig. 1 Examples of some contact forces
l There are circumstances, in which two objects exert forces on one another even though they
are not touching. Such forces are called non-contact forces. Some examples of non-contact
forces are (i) gravitational force, the force between two masses, or an object pulled towards the
D\Allen-Junior wing\Physics\IX\Unit-1\2-Force & Laws of Motion\2019-20

earth due to force of gravity [see fig.2(a)] (ii) electric force, that one electric charge exerts on
another [see fig.2(b)] (iii) magnetic force, exerted by a bar magnet on a piece of iron or on
another magnet [see fig.2(c)].

(a) Gravitational force between (b) Electric force between (c) Magnetic force between
two masse two charges a magnet and a piece of iron
Fig. 2 Examples of non-contact forces
49
Ka
vi
Class IX : Physics

ra
2.0 EFFECTS OF FORCE

cl
as
(1) A force can distort an object i.e. it can change the shape and size of an object.
(2) A force can start an object at rest i.e. it can move a stationary object.

se
(3) A force can stop a moving object i.e. it can cause a moving object comes to rest.

s
(4) A force can change the speed or the magnitude of velocity of an object i.e. it can increase or

de
decrease the speed of an object.

lh
(5) A force can change the direction of a moving object.

i
A force can be defined as ‘a push or pull exerted on an object that can cause the object to speed
up, slow down, or change direction as it moves or it can change its shape and size’.

Fig. 3 Effects of force

ACTIVE PHYSICS
1. Fig.4 shows a wooden block on a horizontal table. Two strings X and Y are tied to the two
opposite faces of the block as shown. Apply a force by pulling the string X, the block begins to
move to the right. Similarly, if we pull the string Y, the block moves to the left.
2. Now, pull both strings X and Y simultaneously i.e. in opposite direction. If the forces on the
block are equal, the block will not move. If the opposite forces on the block are of different
magnitudes, the block will move in the direction of the greater force.
Conclusion : When a single force is applied on an object, the object will move in the direction
of applied force. When two or more forces are applied on an object, the object will move in the
direction of net force acting on the object. Thus, force is a vector quantity.
D\Allen-Junior wing\Physics\IX\Unit-1\2-Force & Laws of Motion\2019-20

Fig. 4 Active physics


50
Ka
vi
ra
Force and Laws of Motion

3.0 BALANCED AND UNBALANCED FORCES

cl
as
3.1 Balanced Forces

se
If the resultant of all forces acting on a body is zero, the forces are called ‘balanced forces’.

s
If the net force exerted on an object is zero, then the forces acting on it are said to be balanced. In

de
such a case, the acceleration of the object is zero and its velocity remains constant. That is, if the
net force acting on the object is zero, the object either remains at rest or continues to move with

lh
constant velocity.

i
3.2 Unbalanced Forces
If the resultant of all forces acting on a body is not zero, the forces are called ‘unbalanced forces’.
If the net force exerted on an object is not zero, then the forces acting on it are said to be unbalanced.
In such a case, the acceleration of the object is not zero and its velocity changes. That is, if the net
force acting on the object is not zero, then such a force changes state of rest or the state of uniform
motion of the object.
4.0 GALILEO'S INCLINED PLANES
Galileo studied motion of objects on an inclined plane. He noted that balls rolling down [see fig.5(a)]
the inclined planes picked up speed (i.e. acceleration), while balls rolling on [see fig.5(b)] up the
inclined planes lost speed (i.e. retardation). From this he reasoned that balls rolling on [see fig.5(c)]
a horizontal plane would neither speed up nor slow down. The ball would finally come to rest not
because of its ‘nature’ but because of friction. This idea was supported by Galileo's observation of
motion along smoother surfaces. When there was less friction, the motion of objects persisted for a
longer time. The smaller the friction, the more the motion approached constant velocity. Galileo
concluded that an object moving on a frictionless horizontal plane must neither have acceleration
nor retardation, i.e. it should move with constant velocity.

Fig. 5

(a) A ball rolling down the plane. Velocity (b) A ball rolling up the plane. (c) A ball rolling on a horizontal
of ball increases (accelerated motion). Velocity of plane ball decreases Velocity of ball remains
(retarded motion). constant (uniform motion)
D\Allen-Junior wing\Physics\IX\Unit-1\2-Force & Laws of Motion\2019-20

Another experiment by Galileo leading to the same conclusion involves a double inclined plane.
A ball released from rest on one of the planes rolls down and climbs up the other. If the planes are
smooth, the final height of the ball is nearly the same as the initial height (a little less but never
greater). In the ideal situation, when friction is absent, the final height of the ball is the same as its
initial height. If the slope of the second plane is decreased and the experiment is repeated, the ball
will still reach the same height, but in doing so, it will travel a longer distance. Galileo concluded
that ‘a ball rolling down the first inclined plane on the left tends to roll up to its initial height on the
second plane on the right, thus, the ball must roll a greater distance as the angle of the second
inclined plane on the right is reduced’. He argued that when the slope of the second plane is made
zero i.e. it becomes a horizontal plane, the ball must travel an infinite distance since it can never
reach its initial height on first plane. In other words, its motion never ceases. This is, of course, an
idealised situation.
51
Ka
vi
Class IX : Physics

ra
cl
(a) (b) (c)

as
se
(Fig.6)

s
Galileo arrived at a new insight that ‘the state of rest and the state of uniform motion (motion with

de
constant velocity) are equivalent’. In both cases, there is no net force acting on the body. It is

lh
incorrect to assume that a net force is needed to keep a body in uniform motion. To maintain a

i
body in uniform motion, we need to apply an external force to counter the frictional force, so that
the two forces sum up to zero net external force. A body does not change its state of rest or uniform
motion, unless an external force compels it to change that state. The tendency of things to resist
changes in motion was what Galileo called inertia.

Illustration 1. What happens when several forces act simultaneously on an object ? What can be the
effect of such forces on the state of motion of the object ?
Solution When several forces act simultaneously on an object, the net force acting on the object
is the vector sum of all forces acting on the object. If the net force acting on the object
is zero, then the body will not accelerate i.e. it may be at rest or in uniform motion. The
object accelerates only if the net force acting on it is not equal to zero.

Illustration 2. What happens when some children try to push a box on a rough floor ?
Solution If they push the box with a small force, the box does not move because of friction acting
in a direction opposite to the push [see fig.(a)]. This frictional force arises between two
surfaces in contact. Here, the friction is between the bottom of the box and floor’s
rough surface. It balances the pushing force and therefore the box does not move. If the
children push the box harder [see fig.(b)] , the box still does not move. This is because
the frictional force still balances the pushing force. If the children continue to increase
the push force on the box, at some point the pushing force becomes bigger than the
frictional force [see fig.(c)]. That is, there is an unbalanced force due to which the
box starts moving (accelerating).

D\Allen-Junior wing\Physics\IX\Unit-1\2-Force & Laws of Motion\2019-20

Objective Questions
1. Which of the following is correct :
(A) Balanced forces may deform a body
(B) Balanced forces may accelerate a body
(C) Balanced forces may move a body in a circle
(D) None of these

2. On drawing water from a well, a force of __________ is applied on the rope :


(A) pull (B) push (C) push or pull (D) none of these

52
Ka
vi
ra
Force and Laws of Motion
3. Muscular force is a :

cl
(A) contact force (B) non contact force (C) any of two (D) none of these

as
se
4. A person is standing on a table, which force is acting :
(A) Unbalanced force (B) Balanced force (C) Both A and B (D) Can’t say

s
de
5. A force cannot :

lh
(A) Set a stationary object in motion (B) Stop a moving object

i
(C) Change the shape of object (D) None of these

Subjective Questions
6. What happens if we stop paddling while riding a bicycle?

7. Is it correct to say that ‘an object maintains its motion under the continuous application of an
unbalanced force’?

8. Which type of force is required to accelerate or retard the motion of an object ?

9. What Aristotelian idea did galileo demolish with his experiments with inclined planes?

10. Define force. What do you understand by a resultant force ?


D\Allen-Junior wing\Physics\IX\Unit-1\2-Force & Laws of Motion\2019-20

53
Ka
vi
Class IX : Physics

ra
FORCE AND LAWS OF MOTION

cl
as
(NLM-2)

se
s
5.0 NEWTON'S FIRST LAW OF MOTION

de
Newton built on Galileo’s ideas and laid the foundation of mechanics in terms of three laws of

lh
motion that go by his name. Galileo’s law of inertia was his starting point which he formulated as
the first law of motion

i
‘Every object continues in its state of rest, or of uniform motion in a straight line, unless it is compelled
to change that state by forces impressed upon it’.
5.1 Concept of Inertia
Inertia is ‘the natural tendency of an object to remain at rest or in motion at a constant speed along
a straight line’. In other words ,‘the tendency of an object to resist any attempt to change its velocity’
is called inertia.
The mass of an object is a qualitative measure of inertia. More the mass, more will be the inertia of
an object and vice-versa.
Inertia of an object can be of three types :
(1) Inertia of rest (2) Inertia of motion (3) Inertia of direction
5.2 Inertia of Rest
It is the tendency of an object to remain at rest. This means an object at rest remains at rest until a
sufficiently large external force is applied on it.
l Examples of inertia of rest
(1) When you are sitting in a stationary car, if the car starts suddenly i.e. accelerates forward, you
feel as if our body is being pushed back against the seat, because your body which was initially
at rest resists this change due to inertia. The lower part of body comes in motion as it is in
direct contact with the car floor, while the upper portion still remains at rest due to inertia of rest
[see fig.7(a)]. If the speed of car increases slowly, you will not feel a push or a jerk because
the inertia of motion will get transferred to the whole body.
(2) When you move a hammer with loose hammerhead in downward motion and suddenly stop it
on a floor or a wooden base, the hammerhead gets tightened. This is because the handle of the
hammer comes in motion while the hammerhead still remains at rest due to inertia of rest [see
fig.7(b)]. If you move the hammer slowly, the inertia of motion will get transferred to the hammer
handle as well as the hammerhead, thus, the hammerhead will not get tightened.
(3) When a blanket is given a sudden jerk, the dust particles fall off. This is because the blanket
suddenly comes in motion but the dust particles still remain at rest. As a result, the dust particles get
separated from the blanket.
D\Allen-Junior wing\Physics\IX\Unit-1\2-Force & Laws of Motion\2019-20

(a) (b)
Fig. 7 Examples of inertia of rest

5.3 Inertia of Motion


It is the tendency of an object to remain in the state of uniform motion. This means an object in
uniform motion continues to move uniformly until an external force is applied on it.
54
Ka
vi
ra
Force and Laws of Motion
Examples of inertia of motion

cl
l

(1) When you are driving a car and you apply brakes to stop the car suddenly, you feel as if your

as
body is being pushed forward, because your body resists the decrease in speed. The lower part

se
of body comes to rest as it is in direct contact with the car floor, while the upper portion still

s
remains in motion due to inertia of motion [see fig.8(a)]. If you stop the car by decreasing its
speed slowly, you will not feel a push or a jerk because the inertia of rest will get transferred to

de
the whole body.

lh
(2) A person jumping out of a moving train has the tendency to fall forward. This is because on

i
jumping, his feet come to rest as they touch the ground. But, his upper body continues to move
forward due to inertia of motion.
(3) An athlete runs for some distance quickly before taking a long jump. As a result, he takes a
longer jump due to inertia of motion [see fig.8(b)].

Inertia of motion
helps a long jumper

(a) (b)

Fig. 8 Examples of inertia of motion

5.4 Inertia of Direction

It is the tendency of an object to maintain its direction. This means an object moving in a particular
direction continues to move in that direction until an external force is applied to change it.
l Examples of inertia of direction
(1) When your motorcar makes a sharp turn at a high speed, you tend to get thrown to one side.
You tend to continue in straight-line motion due to inertia of direction [see fig.9 (a)].

Direction
of motion

Motorist tends
to continue
D\Allen-Junior wing\Physics\IX\Unit-1\2-Force & Laws of Motion\2019-20

in straight line

Fig.9 (a)
(2) When a wheel rotates at high speeds, the sand particles on the wheel fly tangentially along a
straight line due to inertia of direction [see fig.9(b)].
55
Ka
vi
Class IX : Physics

ra
cl
as
se
s
de
lh
i
Fig.9 (b)

ACTIVE PHYSICS
1. Set a five-rupee coin on a stiff smooth card covering an empty glass tumbler standing on a
table (see fig.10). Give the card a sharp horizontal flick with a finger.
2. If we do it fast then the card moves away, allowing the coin to fall vertically into the glass
tumbler due to its inertia. The inertia of the coin tries to maintain its state of rest even when the
card flies off.

(a) (b)

Fig. 10 Active physics

ACTIVE PHYSICS
1. Make a pile of similar carrom coins on a table (see fig.11). Attempt a strong horizontal hit at the
bottom of the pile using another carrom coin or the striker. If the hit is strong enough, the
bottom coin moves out quickly. Once the lowest coin is removed, the inertia of the other coins
makes them ‘fall’ vertically on the table.
2. This is because the lowest coin comes in motion while the other coins remain at rest due to
inertia. If the hit is not so strong, the inertia of motion is transferred to all the coins, thus, the
coins may fall randomly with or without the actual movement of lowest coin.
D\Allen-Junior wing\Physics\IX\Unit-1\2-Force & Laws of Motion\2019-20

Fig. 11 Active physics

56
Ka
vi
ra
Force and Laws of Motion
6.0 LINEAR MOMENTUM (OR MOMENTUM)

cl
as
The linear momentum of a particle or an object that has a mass ‘m’ moving with a velocity ‘v’ is

se
defined to be the product of the mass and velocity.

s
p = mv

de
lh
Linear momentum is a vector quantity, because it equals the product of a scalar quantity m and a

i
vector quantity v. The direction of linear momentum is ‘the direction along the velocity’.
The linear momentum of a particle is
(i) directly proportional to its mass
(ii) directly proportional to its velocity
l Units of linear momentum
SI unit - kg m/s or kg m s–1
(Another SI unit of momentum is Newton-second or N-s)
C.G.S. unit : g cm/s or g cm s–1

Illustration 1. Momentum of an object is 20 kg ms–1. What will be its momentum if


(a) its mass is doubled but the velocity remains the same ?
(b) the velocity is reduced to (1/3) of its original magnitude but mass remains the
same?
Solution Let the mass of the object be m and its velocity be v.
Initial value of momentum, p = mv = 20 kg m s–1
(a) New mass, m' = 2 m, velocity remains the same i.e. v' = v
New value of momentum, p' = m'v' = (2m)(v) = 2mv = 2 ´ 20 = 40 kg m s–1
(b) New velocity, v' = v/3, mass remains the same i.e., m' = m
New value of momentum, p' = m 'v' = m(v/3) = (mv)/3 = 20/3 = 6.67 kg m s–1

Illustration 2. A 65-kg girl is driving a 535-kg car at a constant velocity of 11.5 m/s. Calculate the
momentum of the girl-car system.
Solution Since, we have to find the momentum of the girl-car system, the total mass of the
system,
m = mass of girl + mass of car = 65 + 535 = 600 kg
D\Allen-Junior wing\Physics\IX\Unit-1\2-Force & Laws of Motion\2019-20

Velocity of car, v = 11.5 m/s


Now, momentum, p = mv = 600 ´ 11.5 = 6900 kg m s–1

57
Ka
vi
Class IX : Physics

ra
cl
as
se
Objective Questions

s
1. A body moving with constant velocity :
(A) Has non zero acceleration (B) May have an unbalanced force acting on it

de
(C) May have balanced force acting on it (D) None of these

lh
i
2. A body of mass m collides with a wall with velocity v, and rebounds back with same velocity. What
is the change in momentum of body?
(A) mv (B) 2 mv (C) Zero (D) None

3. A man sitting in a train in motion is facing the engine. He tosses a coin up, the coin falls ahead of
him. The train is moving
(A) forward with uniform speed (B) backward with uniform speed
(C) forward with acceleration (D) forward with deceleration

4. Momentum is a measure of :
(A) Weight (B) Mass (C) Quantity of motion (D) Velocity

5. Momentum is a :
(A) Vector quantity (B) Scalar quantity
(C) Neither scalar nor vector (D) Can be scalar or vector

Subjective Questions

6. What happens when you shake vigorously a branch of a mango tree ?

7. Why a bullet shot on a tightly fitted window glass sheet makes a small circular hole, while a
cricket ball damages a big portion of the same sheet ?

8. A car and a truck both are moving with same velocities. Which one has more momentum ?

9. A car and a truck have same momentum. Which one has more velocity ?

10. The momentum of a 75-g bullet is 9 kg m/s. What is the velocity of the bullet ?
D\Allen-Junior wing\Physics\IX\Unit-1\2-Force & Laws of Motion\2019-20

58
Ka
vi
ra
Force and Laws of Motion

FORCE AND LAWS OF MOTION

cl
as
(NLM-3)

se
s
7.0 NEWTON'S SECOND LAW OF MOTION

de
lh
According to the second law of motion, ‘the rate of change of momentum of a body is directly

i
proportional to the applied force and takes place in the direction in which the force acts’.
Dp
Mathematically, the force, F µ
t
where, Dp = change in momentum, t = time interval.

l Mathematical formulation of second law of motion


Let an object of mass ‘m’ is moving along a straight line with an initial velocity ‘u’. It is uniformly
accelerated to velocity ‘v’ in time ‘t’ by the application of a constant force F throughout the time t.
The initial and final momentum of the object will be, p1 = mu and p2 = mv, respectively.
The change in momentum , Dp = p2 – p1 = mv – mu = m (v – u)
The force F is proportional to the rate of change of momentum, that is,
Dp m(v - u)
Fµ or F µ
t t
m(v - u)
or F=k ...(1) where, k is a constant of proportionality.
t
(v - u)
Now, acceleration, a = ...(2)
t
From eq.(1) & eq.(2), we get, F = k m a ...(3)
The SI units of mass and acceleration are kg and m s–2 respectively. The unit of force is so chosen
that the value of the constant ‘k’ becomes one.
That is, 1 unit of force = k ´ (1 kg) ´ (1 m s–2) or k = 1
Thus, the value of k becomes 1. Therefore, the eq.(3) reduces to,

F=ma

l Unit of force
SI unit : Newton (N)
Since, F = ma
\ 1 N = 1 kg ´ 1 m s–2 = 1 kg m s–2
D\Allen-Junior wing\Physics\IX\Unit-1\2-Force & Laws of Motion\2019-20

C.G.S unit : Dyne


1 dyne = 1 g cm s–2
1 N = 1000 g ´ 100 cm s–2 = 105 g cm s–2 , or 1 N = 105 dynes
1
If force is constant i.e., F = ma = constant, then, the acceleration produced in the body, a µ .
m
That is, for a given force, acceleration produced is inversely proportional to its mass.
If same force F is applied to masses m1 and m2 and the resulting accelerations in them are a1 and
a2 respectively, then,
a2 m1
m1a1 = m2a2 or =
a1 m2

59
Ka
vi
Class IX : Physics

ra
7.1 Applications / Results of Newton’s Second Law

cl
(i) Suppose a light-weight vehicle (say a small car) and a heavy-weight vehicle (say a loaded

as
truck) are parked on a horizontal road. A much greater force is needed to push the truck than

se
the car to bring them to the same speed in same time. This is because, for a given time interval,

s
the force is directly proportional to the change in momentum. Here, the change in momentum

de
of truck is quite large than that of the car, therefore, force required for truck is quite large as
compared to that required for car.

lh
(ii) An experienced cricketer catches a cricket ball coming in with great speed far more easily than a

i
beginner, who can hurt his hands in the act. One reason is that the cricketer allows a longer time for
his hands to stop the ball. He draws in the hands backward in the act of catching the ball . As the
time for catching increases, the force with which the ball hurts his hand decreases. As a result, his
hands are not injured. A beginner, on the other hand, keeps his hands fixed and tries to catch the
ball almost instantly (i.e. in a very small time interval). Thus, a much greater force is exerted by the
ball on his hands and this hurts a lot.
(iii) If two stones, one light and the other heavy, are dropped from the top of a building, a person on
the ground will find it easier to catch the light stone than the heavy stone. This is because the
force is directly proportional to the mass of an object.
(iv) Speed/velocity is another important parameter to consider. A bullet fired by a gun can easily
pierce human tissue before it stops, resulting in casualty. The same bullet fired with moderate
speed will not cause much damage. Thus for a given mass, the greater the speed, the greater is
the opposing force needed to stop the body in a certain time.
(v) When an athlete goes for a high jump, he is made to fall on a cushioned bed. This increases the
time of falling of the athlete, thereby reducing the force exerted on him, causing no injury.

8.0 IMPULSE OF A FORCE (J)

mv - mu
We have F=
t
Þ Ft = mv – mu … (i)
The quantity Ft is called the impulse of the force F in the time interval t. If we equate t2 – t1 to t, the
impulse is expressed as
J = Ft … (ii)
From equation (i) we see that the impulse of a force acting on a body is equal to the change in the
linear momentum of the body produced by the force.
Quite often, we have to work with forces which have very large magnitudes but which act for very
small time intervals. For example, when a ball rebounds from a hard floor, it remains in contact
with the floor for very short time, The floor exerts a large force on the ball in this short time. Such
a force is called an impulsive force. Although the force is very large and the time is very short, their
product (the impulse) remains finite. Thus, an impulsive force produces a finite change in
D\Allen-Junior wing\Physics\IX\Unit-1\2-Force & Laws of Motion\2019-20

momentum.

Illustration 1. Equal forces are applied to a car and a truck which produce acceleration in both of them.
Which one has smaller acceleration ?
Solution For a given force, the acceleration produced in the body is inversely proportional to the
mass of the object (a µ 1/m). This means more the mass of an object, less will be the
acceleration produced init. Here, force applied on the truck as well as the car is same.
Since, the mass of a truck is greater than the mass of a car, therefore, the acceleration
of truck is smaller than the acceleration of car.
60
Ka
vi
ra
Force and Laws of Motion
Illustration 2. Estimate the net force needed to accelerate a 1000 kg car at 5 m/s2. If same force is

cl
applied to a 200 g apple, what will be its acceleration ?

as
Solution Given, mass of car, m = 1000 kg

se
acceleration of car, a = 5 m/s2

s
force needed, F = ?
Force, F = ma = 1000 ´ 5 = 5000 N

de
Now, same force is applied to a 200 g apple i.e., F = 5000 N

lh
mass of apple, m' = 200 g = 200/1000 = 0.2 kg

i
F 5000
Thus, acceleration of apple, a' = = = 2.5 ´ 104 m/s2
m' 0.2

Illustration 3. What force is required to bring a 900 kg car to rest from a speed of 120 km/h within a
distance of 50 m?
Solution Given, mass of car, m = 1500 kg ; final speed, v = 0 ;
initial speed, u = 120 km/hr = 120 ´ (5/18) = (100/3) m/s
From third equation of motion,
v2 = u2 + 2as
or (0)2 = (100/3)2 + 2a ´ (50)
100 ´ 100 100
or a = - =- m/s2
3 ´ 3 ´ 2 ´ 50 9
Now, force, F = ma = 900 ´ (–100/9) = –10000 = –1 ´ 104 N

Illustration 4. A constant force acts on an object of mass 5 kg for a duration of 2 s. It increases the
object’s velocity from 3 m s –1 to 7 m s–1. Find the magnitude of the applied force. Now,
if the force was applied for a duration of 5 s, what would be the final velocity of the object?
Solution Given, initial velocity, u = 3 m s–1 ; final velocity, v = 7 m s–1 ;
time, t = 2 s ; mass of the object, m = 5 kg ; force, F = ?
v – u 7–3 4
Now, acceleration, a = = = = 2 m s–2
t 2 2
Force, F = ma = 5 ´ 2 = 10 N
If same force was applied for a duration of 5 s, the acceleration will remain the same i.e.,
a = 2 m s–2
Now, from first equation of motion, v = u + at = 3 + 2 ´ 5 = 3 + 10 = 13 m s–1

Objective Questions
D\Allen-Junior wing\Physics\IX\Unit-1\2-Force & Laws of Motion\2019-20

1. When force of 1N acts on mass of 1kg., which is placed on a frictionless horizontal surface, the
object moves with :
(A) constant speed of 1 ms–1 (B) constant speed of 1 kms -1
(C) acceleration of 10 ms-2 (D) acceleration of 1ms-2

2. A stationary ball weighing 0.25 kg acquires a speed of 10 m/s when hit by a hockey stick. The
impulse imparted to the ball is :
(A) 2.5 N s (B) 2.0 N s (C) 1.5 N s (D) 0.5 N s

61
Ka
vi
Class IX : Physics

ra
3. Choose correct statement :

cl
(A) Impulse is the total change in momentum produced during impact

as
(B) Impulse is product of distance and time

se
(C) Impulse is force acting per unit length

s
(D) None of these

de
lh
4. Force measures the rate of change of _______ of a body :

i
(A) Mass (B) Inertia
(C) Velocity (D) Momentum

5. In the figure shown, acceleration of the body is :


5N
10N 2 Kg 6N

(A) 2 m/s2 (B) 4.5 m/s2 (C) 10.5 m/s2 (D) 7.5 m/s2

Subjective Questions
6. A greater opposing force is needed to stop a heavy body than a light body in the same time, if they
are moving with the same speed. Explain.

7. How seat belts help to prevent injuries to the passengers in car in case of an accident ?

8. The velocity-time graph of a ball of mass 20 g moving along a straight line on a long table is given
in fig. How much force does the table exert on the ball to bring it to rest ?

9. Can you say that no force acts on a body at rest ? or is it correct to say that no net force acts on
it? Defend your answer.
D\Allen-Junior wing\Physics\IX\Unit-1\2-Force & Laws of Motion\2019-20

10. Can an object be in equilibrium when only a single force acts on it? Explain.

62
Ka
vi
ra
Force and Laws of Motion

FORCE AND LAWS OF MOTION

cl
as
(NLM-4)

se
s
9.0 NEWTON'S THIRD LAW OF MOTION

de
The second law relates the external force on a body to its acceleration. What is the origin of the

lh
external force on the body? What agency provides the external force ? The simple answer is that

i
the external force on a body always arises due to some other body.
l Forces always exist in pairs
When two objects interact, two forces will always be involved. One force is the action force and
the other is the reaction force.
According to Newton’s third law, ‘whenever one body exerts a force on a second body, the second
body exerts an oppositely directed force of equal magnitude on the first body’.
In other words, ‘to every action, there is always an equal and opposite reaction’. Consider a pair
of bodies A and B. According to Newton’s third law, FAB = – FBA , where, FAB = force on A due to B, and
FBA= force on B due to A.

9.1 Examples / Applications of Newton’s Third Law


1. If you press a coiled spring, the spring is compressed by the force of your hand. The compressed
spring in turn exerts a force on your hand and you can feel it (see fig.12).

Action : Force on spring by your hand Reaction : Force on your hand by the spring
Fig. 12

2. Newton’s third law is also applicable to non-contact forces. For example, the Earth pulls an object
downwards due to gravity (see fig.13). The object also exerts the same force on the Earth but
in upward direction. But, we hardly see the effect of the object on the Earth because the Earth
is very massive and the effect of a small force on its state of rest or motion is negligible. That
is, the acceleration of Earth is negligible due to its huge mass.
D\Allen-Junior wing\Physics\IX\Unit-1\2-Force & Laws of Motion\2019-20

Fg = Force on object due to Earth


Fe = Force on Earth due to object

Fig. 13 Newton’s third law is also applicable to non-contact forces


63
Ka
vi
Class IX : Physics

ra
3. When we strike a nail using a hammer to fix it in a board, the hammer exerts a downward

cl
force on the nail, and the nail exerts an upward force on the hammer; these forces are equal

as
in magnitude (see fig.14).

se
s
de
lh
i
F1 = Force on hammer due to nail
F2 = Force on nail due to hammer
Fig. 14 In the interaction between the hammer and the nail, each exerts the same amount of force on the other

4. If Ram exerts a small force on the wall, the wall will exert a small force on Ram [see fig.15(a)].
When Ram pushes hard against the wall, it pushes back just as hard [see fig.15(b)].

(a) (b)
Fig. 15
5. Suppose you are standing at rest and intend to start walking (or running) on a road [see fig.16].
While walking, you push the road backwards. Thus, according to Newton’s third law, the road
exerts an equal and opposite reaction force on your feet to make you move forward.
D\Allen-Junior wing\Physics\IX\Unit-1\2-Force & Laws of Motion\2019-20

Fig.16
6. When a gun is fired, it exerts a forward force on the bullet. The bullet exerts an equal and opposite
reaction force on the gun. This results in the recoil of the gun [see fig.17]. Since the gun has
a much greater mass than the bullet, the acceleration of the gun is much less than the acceleration
of the bullet.

64
Ka
vi
ra
Force and Laws of Motion

cl
as
se
s
de
lh
i
Fig. 17
7. When a sailor jumps out of a rowing boat. As the sailor jumps forward, the force on the boat
moves it backwards [see fig.18].

Fig. 18

8. Consider an inflated balloon recoiling when air is expelled [see fig.19]. When the air is expelled
leftward, the balloon accelerates rightward.

Fo rce exer ted


by ex haust gas
o n ro cket

Fig. 19
9. Rocket Propulsion - In a rocket engine, the highly combustible fuel burns
at a tremendous rate (see fig.20). The rocket exerts a downward (or
backward) force on the exhaust gas and thus, according to Newton’s
third law, the exhaust gas exerts an upward (or forward) force on the
rocket; these forces are equal in magnitude. It is the reaction force of
D\Allen-Junior wing\Physics\IX\Unit-1\2-Force & Laws of Motion\2019-20

the exhaust gas that accelerates the rocket forward. That is why a rocket
can accelerate even in outer space.

Forc e exer ted


by rocket o n
exh aust gas

Fig. 20
65
Ka
vi
Class IX : Physics

ra
10.0 CONSERVATION OF MOMENTUM

cl
The second and the third laws of motion lead to an important result : the law of conservation

as
of momentum. Let us understand it by taking an example : When a bullet is fired from a gun,

se
according to Newton’s third law, if the force on the bullet by the gun is F, then the force on the gun

s
by the bullet is – F. The two forces act for a common interval of time t.

de
According to Newton’s second law, force F can be written as,

lh
Dp

i
F= where, Dp is the change in momentum of the object.
t
or Dp = F ´ t
This means, F ´ t is the change in momentum of the bullet and – F ´ t is the change in momentum
of the gun. Since initially, both are at rest, the change in momentum equals the final momentum
for each. Let pb be the momentum of the bullet after firing and pg is the recoil momentum of the
gun, then,
pb = F ´ t ... (1) and pg = – F ´ t ... (2)
Adding eq.(1) and eq.(2), we get, pb + pg = 0. That is, the final momentum of the system (bullet
plus gun) is zero. But, initial momentum of the system is also zero. This means initial momentum
is equal to the final momentum i.e., total momentum is conserved.
Thus, in an isolated system (a system with no external force), mutual forces between pairs of particles
in the system can cause momentum change in individual particles, but since the mutual forces for
each pair are equal and opposite, the momentum changes cancel in pairs and the total momentum
remains unchanged. This fact is known as the law of conservation of momentum.
According to law of conservation of momentum, ‘when the net external force on a system of objects
is zero, the total momentum of the system remains constant’.

l Law of conservation of momentum - proof


Let us consider two balls A and B having masses m A and mB, travelling in the same direction along
a straight line at different velocities uA and uB, respectively [see fig.21(a)]. No other external
unbalanced forces are acting on them. Let uA > uB and the two balls collide with each other as
shown in fig.21(b). During collision which lasts for a very short time t, the ball A exerts a force F BA
on ball B, and the ball B exerts a force FAB on ball A. Suppose vA and vB are the velocities of the
two balls A and B after the collision, respectively [see fig.21(c)].

Fig. 21
D\Allen-Junior wing\Physics\IX\Unit-1\2-Force & Laws of Motion\2019-20

Initial momentum of ball A = mAuA, and final momentum of ball A = mAvA


Force on A due to B, FAB = rate of change of momentum of ball A
m A v A - m A u A m A (v A - u A )
or FAB = = ...(1)
t t
Initial momentum of ball B = mBuB, and final momentum of ball B = mBvB
Force on B due to A, FBA = rate of change of momentum of ball B
mB v B - mB uB mB (v B - uB )
or FBA = = ...(2)
t t
According to Newton’s third law of motion, the force FBA exerted by ball A on ball B (action) and
the force FAB exerted by the ball B on ball A (reaction) must be equal and opposite to each other.
66
Ka
vi
ra
Force and Laws of Motion
That is, FBA = – FAB

cl
mB (v B – uB ) m (v – uA )

as
=– A A
t t

se
or mB(vB – uB) = – mA(vA – uA)

s
or mBvB – mBuB = – mAvA + mAuA

de
or mBvB + mAvA = mBuB + mAuA ...(3)

lh
Since, (mBvB + mAvA) represents total final momentum and (mBuB + mAuA) represents total initial

i
momentum, from eq.(3), we can conclude that
Total final momentum = Total initial momentum
This is law of conservation of momentum.

ACTIVE PHYSICS
1. Take a big rubber balloon and inflate it fully. Tie its neck using a thread. Also using adhesive
tape, fix a straw on the surface of this balloon. Pass a thread through the straw and hold one
end of the thread in your hand or fix it on the wall (see fig.22). Fix the other end of the thread
on a wall at some distance. Now remove the thread tied on the neck of balloon. Let the air
escape from the mouth of the balloon.
2. You will observe that the air escapes from the balloon in backward (leftward) direction while
the balloon along with the attached straw moves in forward (rightward) direction. This is in
agreement with Newton’s third law or the law of conservation of momentum.

Fig. 22 Active physics

Illustration 1. A bullet of mass 20 g is horizontally fired with a velocity 150 m s –1 from a pistol of mass
2 kg. What is the recoil velocity of the pistol?
Solution Given, mass of bullet, m1 = 20 g = 0.02 kg ; mass of pistol, m2 = 2 kg;
initial velocity of bullet, u1 = 0 ; initial velocity of pistol, u2 = 0.
Let the direction of bullet is taken left to right. Now, the final velocity of the bullet,
v1 = + 150 m s–1 (by sign convention, left to right is taken positive).
Let ‘v’ be the recoil velocity of the pistol.
D\Allen-Junior wing\Physics\IX\Unit-1\2-Force & Laws of Motion\2019-20

Initial momentum of the pistol-bullet system,


pi = initial momentum of the pistol + initial momentum of the bullet
= 2 ´ 0 + 0.02 ´ 0 = 0 kg m s–1
Final momentum of the pistol-bullet system,
pf = final momentum of the pistol + final momentum of the bullet
= 2 ´ v + 0.02 ´ 150 = (2v + 3) kg m s–1
According to the law of conservation of momentum,
final momentum of the system = initial momentum of the system
\ 2v + 3 = 0 or 2v = –3 or v = –3/2 = –1.5 m/s
(Negative sign indicates that the direction in which the pistol would recoil is opposite
to that of bullet, that is, right to left).
67
Ka
vi
Class IX : Physics

ra
Illustration 2. A girl of mass 40 kg jumps with a horizontal velocity of 5 m s–1 onto a stationary cart

cl
with frictionless wheels. The mass of the cart is 3 kg. What is her velocity as the cart

as
starts moving ? Assume that there is no external unbalanced force working in the

se
horizontal direction.

s
Solution Given, mass of girl, m1 = 40 kg ; initial velocity of girl, u1 = + 5 m/s ;
mass of cart, m2 = 3 kg ; initial velocity of cart, u2 = 0.

de
Initial momentum of the girl-cart system,

lh
pi = Initial momentum of the girl + Initial momentum of the cart

i
or pi = m1u1 + m2u2 = (40) ´ (+5) + (3) ´ (0) = 200 kg m/s
Let ‘v’ be the common velocity of the girl-cart system when the cart starts moving along with
the girl.
Final momentum of the girl-cart system,
pf = Final momentum of the girl + Final momentum of the cart
or pf = m1v + m2v = (m1 + m2) v = (40 + 3) v = 43 v kg m/s
According to the law of conservation of momentum,
final momentum of the system = initial momentum of the system
or 43 v = 200, or v = 200/43 = + 4.65 m/s
The girl on cart will move with a velocity of 4.65 m/s in the direction in which the girl
jumped.

Illustration 3. Consider a large fish that swims towards and swallows a small fish at rest (see fig.). If
the large fish has a mass of 5 kg and swims at 1 m/s towards the small fish having mass
of 1 kg, what is the velocity of the large fish immediately after the lunch ? Neglect the
effects of water resistance.

Solution Given, mass of large fish, m1 = 5 kg ; initial velocity of large fish, u1 = 1 m/s ;
mass of small fish, m2 = 1 kg ; initial velocity of small fish, u2 = 0.
Let ‘v’ be the velocity of the large fish-small fish system.
Applying law of conservation of momentum, we get,
total momentum after lunch = total momentum before lunch
or m1 v + m2 v = m1u1 + m2u2
or (m1 + m2) v = m1u1 + m2u2
or (5 + 1) v = (5) ´ (+1) + (1) ´ (0)
or 6 v = 5,or v =(5/6) m/s or v = + 0.833 m/s
The large fish will move with a velocity of 0.833 m/s in the direction in which it was
moving initially.
D\Allen-Junior wing\Physics\IX\Unit-1\2-Force & Laws of Motion\2019-20

Illustration 4. If the action force is equal in magnitude to the reaction force, how can there ever be
an acceleration? Explain using an example.
Solution Though the action-reaction pair are equal in magnitude and opposite in direction but
the reaction force always acts on a different object than the action force. Thus, these
forces do not cancel out each other. Hence, there can be an acceleration in an object.
Example : Let us consider a volleyball player bumping the ball (see fig.). At the instant
when both the ball and the player’s arms are in contact, the action force is the upward
force that the player exerts on the ball. The reaction force is the downward force that
the ball exerts on the player’s arms. During the collision, the ball accelerates upward
and the player’s arms accelerate downward. We hardly notice the acceleration of player’s

68
Ka
vi
ra
Force and Laws of Motion
arms since his mass is quite large as compared to the ball, and the effect of the

cl
force on his motion is negligible.

as
se
s
de
lh
i
Illustration 5. A horse pulls a cart along a level road, causing the cart to accelerate, as shown in figure.
Newton’s third law states that the cart exerts an equal and opposite force on the horse.
In view of this, how can the cart accelerate? Under what condition does the horse-cart
system move with constant velocity?

Solution According to Newton’s third law, action-reaction forces act on different objects—the horse
exerts a force (T) on the cart, and the cart exerts an equal magnitude and oppositely
directed force (T) on the horse. But, horse also exerts a force (due to friction) on the
road in backward direction, as a result of which the road exerts a force ‘Fhorse’ on the
horse in forward direction. Thus, the horse starts moving in forward direction. Now, cart
is attached to the horse, thus, the wheels of cart start rotating which exerts a force ‘Fcart’
(due to friction) on the road in backward direction,as a result of which the road also
exerts an equal force on the cart in forward direction. This is how, the horse-cart system
moves. When ‘Fhorse’ is greater than ‘Fcart’, the horse-cart system accelerates. When ‘Fhorse’
is equal to ‘Fcart’, the horse-cart system moves with constant velocity.
D\Allen-Junior wing\Physics\IX\Unit-1\2-Force & Laws of Motion\2019-20

69
Ka
vi
Class IX : Physics

ra
cl
as
se
Objective Questions
1. Action and reaction :

s
(A) Acts on same body (B) Acts on different bodies

de
(C) Any of (A) and (B) (D) Can’t say

lh
2. A bullet of mass 0.01 kg is fired from a gun weighing 5.0 kg. If the initial velocity of the bullet is

i
250 m/s. Calculate the velocity with which the gun recoils :
(A) – 0.50 m/s (B) – 0.25 m/s (C) + 0.05 m/s (D) + 0.25 m/s

3. If a moving ball A collides with another moving ball B, on a frictionless surface, then :
(A) Momentum of A = Momentum of B
(B) (Momentum of A + Momentum of B) before collision = (Momentum of A + Momentum of B)
after collision
(C) Neither (A) nor (B)
(D) (A) and (B) both

4. When a bullet is fired from a gun. The gun recoils to :


(A) Conserve mass (B) Conserve momentum
(C) Conserve K.E (D) None of these

5. Which of the following is not an action-reaction pair?


(A) Force applied by man on earth & force applied by the earth on man while walking
(B) Pushing the water behind by a man & pushing the man forward by water while swimming
(C) Pulling the air from ahead by aeroplane & pushing the aeroplane by air
(D) Weight of a book & reaction force by table

Subjective Questions
6. A high-speed car and an innocent insect have a head-on collision. The force of impact splatters
the poor insect over the windshield. Is the corresponding force that the insect exerts against the
windshield greater, less, or the same? Is the retardation of the car greater than, less than, or the
same as that of the insect ?

7. Take two spring balances connected together as shown in fig. The fixed end of balance B is attached
with a rigid support, like a wall. Apply a force through the free end of spring balance A. You will
observe that both the spring balances show the same readings on their scales. Explain how it is
possible.
D\Allen-Junior wing\Physics\IX\Unit-1\2-Force & Laws of Motion\2019-20

8. Newton's second law states that, if no net force is exerted on a system, no acceleration occurs. Does
it follow that no change in momentum occurs ?

9. Explain, the recoiling of the gun, using law of conservation of momentum.

10. Two ice hockey players of opposite teams, while trying to hit a hockey ball on the ice collide and
immediately become entangled. One has a mass of 60 kg and was moving with a velocity
5.0 m/s while the other has a mass of 55 kg and was moving faster with a velocity 6.0 m/s towards
the first player. In which direction and with what velocity will they move after they become entangled?
Assume that the frictional force acting between the feet of the two players and ice is negligible.

70
Ka
vi
ra
Force and Laws of Motion

FORCE AND LAWS OF MOTION

cl
as
(NLM-5)

se
s
11.0 THE NORMAL FORCE AND TENSION IN THE STRING

de
lh
11.1 Normal Force

i
Let us consider a television set at rest placed on a table (see fig.23). We
know that the gravitational force is acting on the television. Why the
television does not continue to fall towards the center of Earth ? An
analysis of the forces acting on the television will reveal the forces that
are in equilibrium. First, the gravitational force of Earth (or weight), Fg,
is acting downward. Since the television is in equilibrium, there must
be another force acting on it that is equal in magnitude to Fg, but in the
opposite direction. This force is the force exerted on the television by
the table. This force is called the normal force, Fn.
A force that acts on a surface in a direction perpendicular to the surface
is called normal force. Fig. 23

11.2 Tension In The String


Strings are assumed to be inextensible i.e., they cannot be stretched. Due to this assumption
'acceleration of masses connected through a string is always same. They are assumed to be massless
unless it is mentioned. Due to this assumption 'tension in the string is same every where'.
If the string has mass, tension at different points will be different. It is maximum at the end at which
force is applied and minimum at the other end connected to a mass.
M in im u m T M a x im u m T
F F
T = co n stan t

A m a ss le ss strin g A str in g h av ing m as s

Fig.24 Tensions in the string


The direction of tension at body (or a point) is always outward along the string i.e., away from the
body along the string. A tension always have pulling action.
l Free body diagrams
D\Allen-Junior wing\Physics\IX\Unit-1\2-Force & Laws of Motion\2019-20

A system diagram is a sketch of all the objects involved in a situation. A free-body diagram
(FBD) is a drawing in which only the object being analyzed is drawn, with arrows showing all
the forces acting on the object.
1. Free body diagrams represent all forces acting on one object.
2. Forces that the object exerts on other objects do not appear in free body diagrams because they
have no effect on the motion of the object itself.
3. In drawing a free body diagram, you can represent the object as a single dot or a simplified
shape the object.

71
Ka
vi
Class IX : Physics

ra
4. In FBD each force acting on the object is represented with an arrow. The arrow’s direction

cl
shows the direction of the force and the arrow’s relative length provides information about the

as
magnitude of the force.

se
5. Forces that have the same magnitude should be sketched with approximately the same length,

s
forces that are larger should be longer, and smaller forces should be shorter.

de
6. In case of objects in motion, the direction of acceleration should be made on the FBD in the

lh
direction of greater force (or net force).

i
Fig.25 Making free body diagrams

l Motion of bodies connected by strings


Let us consider two bodies m1 and m2 placed on horizontal frictionless plane connected by a
massless string. Let the mass m1 is pulled by a force F. As a result the whole system moves in the
direction of applied force with an acceleration a. Let the tension in the string be T (see fig.26).

Fig.26 Motion of bodies connected by strings


D\Allen-Junior wing\Physics\IX\Unit-1\2-Force & Laws of Motion\2019-20

For mass m1, F – T = m1 a ....(1) [ F is greater force as it is in the direction of acceleration a]


For mass m2, T = m2 a ....(2) [Here, T is the only force acting on m2]
(1) + (2) Þ (F – T) + T = m1a + m2a

F
or F = (m1 + m2)a or a=
m1 + m 2

m 2F
From (2), we have, T = m2a \ T=
m1 + m 2

72
Ka
vi
ra
Force and Laws of Motion

Motion of bodies connected by string passing over a light pulley (Atwood’s Machine)

cl
l

Let us consider two masses m1 and m2 passing over a light pulley connected through a string (see

as
fig.27). The term 'light pulley' means the mass of pulley is neglected, it is assumed to be massless.

se
Since the two bodies are connected with each other, both move with same acceleration a. Let

s
m2 > m1 then, m2 will go downwards while m1 will go upwards.

de
For m1, T – m1g = m1a ....(1)

lh
[Here, T > m1g, as T is in the direction of acceleration a]

i
For m2, m2g – T = m2a ....(2)
T
[Here, m2 g > T, as m2g is in the direction of acceleration a] T

(1) + (2) Þ (T – m1g) + (m2g – T) = m1a + m2a T a


a m2
a T m1
or (m2 – m1)g = (m2 + m1)a
m1 a m 1g m 2g
(m 2 - m1 )g
or a = (m + m ) m1g FB D for m 1 FBD for m 2
2 1 m2

(Since a ¹ g, two bodies are not free falling bodies.) m 2g


Fig.27 M otion of m asse s connected by a pulle y
2m1m 2g
Putting the value of a in eq.(1), we get, T = m + m
1 2

l Motion of bodies in contact


Let two bodies of masses m1 and m2 respectively are placed side by side touching each other. A
push force 'F' is applied on m1 such that both the bodies start moving together with an acceleration
'a'. Since both the bodies are touching each other there is a pair of action reaction force between
them at place of their contact. These forces are called normal contact forces (see fig.28) and obviously
they are equal in magnitude but opposite in direction (Newton's third law).

Fig.28 Motion of bodies in contact


D\Allen-Junior wing\Physics\IX\Unit-1\2-Force & Laws of Motion\2019-20

For mass m1, F – f = m1 a ....(1) [ F is greater force as it is in the direction of acceleration a]


For mass m2, f = m2 a ....2) [Here, f is the only force acting on m2]
(1) + (2) Þ (F – f) + f = m1a + m2a

F
or F = (m1 + m2)a or a=
m1 + m 2

m 2F
From (2), we have, f = m2a \ f=
m1 + m 2

73
Ka
vi
Class IX : Physics

ra
Weight of an object in a lift

cl
l

A weighing machine measures the normal force not the 'true weight'. Thus, if the normal force

as
changes, the weighing machine does not give reading of true weight, it gives a reading of normal

se
force which we can be called 'apparent weight' of the object.

s
Let us consider a girl standing in a lift.

de
lh
i
Fig.29 Weight of an object in a lift
1. When the lift is at rest or in uniform motion, net acceleration of the system is zero [see fig.29(a)].
Thus, net force on it is zero.
\ Net force, Fnet = mg – R = 0 or R = mg
The R represents the apparent weight, i.e., W = R = mg [Apparent weight = true weight]
2. When let the lift is moving up with uniform acceleration a [see fig.29(b)]. Thus, net force on it
is not zero.
\ Net force, Fnet = R' – mg = ma
[R' is greater force as it is in the direction of acceleration a]
or R' = ma + mg = m (a + g)
The R' represents the apparent weight, i.e., W' = R' = m(a + g) [Apparent weight > true
weight]
3. When let the lift is moving down with uniform acceleration a [see fig.29(c)]. Thus, net force on
it is not zero.
\ mg – R'' = ma [mg is greater force as it is in the direction of acceleration a]
or R'' = mg – ma = m (g – a)
The R'' represents the apparent weight, i.e., W'' = R'' = m(g – a)
[Apparent weight < true weight]
Suppose the rope of the lift breaks, then it will fall freely under gravity i.e., a = g. In this
situation, apparent weight, W'' = R'' = m(g – g) = 0. That is, the weighing machine will read
D\Allen-Junior wing\Physics\IX\Unit-1\2-Force & Laws of Motion\2019-20

zero weight.

Illustration 1. Two masses m1 = 3kg and m2 = 4kg are connected to the


two free ends of an inextensible string which passes over a
frictionless pulley suspended by an inextensible, string to a
rigid support as shown in fig. Calculate (i) the acceleration
of the system, (ii) tension in the string used for connecting
the masses and (iii) the tension in the string used for m1=3kg
m2=4kg
suspending the pulley.

74
Ka
vi
ra
Force and Laws of Motion

cl
æ m 2 – m1 ö
Solution. (i) acceleration, a = ç m +m ÷ g

as
è 1 2ø

se
(4 – 3)
or a= × 9.8 ms–2

s
(4 + 3)

de
9.8
= = 1.4 ms–2

lh
7

i
(ii) Tension in the string used for connecting the masses,
æ 2m1m 2 ö
T = çè m + m ÷ø g
1 2

æ 2× 3× 4 ö 24 ×9.8
= ç ÷ ×9.8 = = 33.6 N
è 3+ 4 ø 7
(iii) Tension in the string used for suspending the pulley,
T1 = T + T = 2T = 2×33.6 = 67.2 N

Illustration 2. Calculate the acceleration of each block and the tension in the string as shown in fig.
Assume, there is no force of friction between the surface of block and the surface of table.

m1

m2

Solution. 1. Mark block of mass m1 and mass m2 as points.

2. Forces acting on the block of mass m1 are (i) its own weight (W1 = m1g) in the
vertically downward direction.
(ii) Normal reaction (R) of the table on the block in the vertically upwards direction
and (iii) The tension (T) in the string joining the two blocks as shown in figure.
3. Forces acting on the block of mass m2 are (i) its own weight (W2 = m2g) in the
vertically downward direction and (ii) the tension (T) in the string joining the two
D\Allen-Junior wing\Physics\IX\Unit-1\2-Force & Laws of Motion\2019-20

blocks as shown in figure.


4. Let a be the acceleration of each block such that block of mass m1 moves horizon-
tally over the surface of the table and to the right side. Block of mass m2 moves with
an acceleration a in the vertically downward direction.
5. Since block of mass m1 moves horizontally, so the net force acting on the block in
the vertical direction is zero i.e., W1 = R and net force acting on the block in the
horizon tal direction = T or F = m1 a = T ...(i)
6. Net force acting on the block of mass m2 in the downward direction,
F = W2 – T = m2 g – T
or m2 a = m2 g – T ...(ii)

75
Ka
vi
Class IX : Physics

ra
7. Adding eqns. (i) and (ii), we get

cl
m1a + m2a = m2g

as
mg

se
or a = (m +m )
2
...(iii)

s
1 2

8. Substituting the value of eqn. (iii) in eqn. (i), we get

de
m2 g m1m1g
T = m1 ´ =

lh
(m1 +m2 ) (m1 +m2 ) ...(iv)

i
Illustration 3. A block of mass 2kg hanged with a rope is going upward with an acceleration of 2 ms–
2
. Calculate the tension in the rope.
Solution The equation of motion of the block is given by T
F = T – mg
or ma = T – mg a 2kg
i.e. T = mg + ma = m(g + a)
= 2(9.8 + 2) mg
= 2 ×11.8 = 23.6 N

Illustration 4. A person holding a box of 30 kg on his head is standing on a smooth surface. Calculate
the force with which the surface pushes his feet. Given, mass of the person is 70 kg.
Solution According to Newton's third law of motion, the force acting on the feet of the person. F
= Total force acting downward on the floor
= weight of the person + weight of the box
= Mg + mg = (M + m) g
= (70 + 30) × 9.8 = 100 × 9.8 = 980 N

Illustration 5. Three blocks are connected as shown in figure on a horizontal frictionless table and
pulled to the right with a force F = 50 N. If m1 = 5 kg, m2 = 10 kg and m3 = 15 kg, find
the tensions T1 and T2.
Solution
a

T1 m2 T2 m3 F
m1

All the three blocks move with an acceleration a due to the force F = 50 N.
\ F = (m1 + m2 + m3)a
F
or a = m + m + m
D\Allen-Junior wing\Physics\IX\Unit-1\2-Force & Laws of Motion\2019-20

1 2 3

F 5
= = ms–2
5+10+15 3
Determination of T2
The force F acts on mass m3 to the right side, whereas tension T 2 acts on mass m3 to the
5
left. Since motion is towards the right side, therefore, 50 – T2 = 15× = 25
3
T2 = 25 N and
5 25
T1 = 5 ´ N Þ N
3 3

76
Ka
vi
ra
Force and Laws of Motion

cl
as
Objective Questions

se
1. If m1 = 20 kg & m2 = 10 kg find normal force acting on the floor m2

s
as shown in fig.?

de
m1
(A) 300 N (B) 100 N

lh
(C) 200 N (D) None of these

i
[ g = 10m/s2 ]

2. Consider the situation shown in figure. The wall is smooth but the
surfaces of A and B in contact are rough. The friction of B due to A
in B A F
equilibrium.
(A) is upward (B) is downward
(C) is zero (D) the system cannot remain in equilibrium.

3. Find the tension T2 in the system shown in Fig. :


T1 1 kg
(A) 1g N (B) 2g N T2
(C) 5g N (D) 6g N 2 kg
T3
3 kg

4. If a string is inextensible the magnitude of acceleration of any number of masses connected through
string is :
(A) Always same (B) Always different (C) Sometime different (D) None of these

5. An object of mass 30 kg is falling in air and experiences a force due to air resistance of 50 N.
Determine the Net force acting on the object ? (take g = 9.8 ms–2)
(A) 122 N (B) 50 N (C) 244 N (D) 100 N

Subjective Questions
6. Find acceleration and contact force between the two bodies as shown in fig.
100N 30
20

7. A system consisting of two blocks as shown in figure moves over a horizontal smooth surface when
a horizontal force of 10 N is applied on it. Find the tension in the connecting string and the accel-
eration of the system. Neglecting friction.
M 1=2kg M2=2kg
D\Allen-Junior wing\Physics\IX\Unit-1\2-Force & Laws of Motion\2019-20

F=10N

8. A body of mass 30 kg stands on a weighing machine lying on the floor of a lift. What will be the
weight of the body when (i) lift moves upwards with an acceleration = 2.2 ms–2, (ii) lift moves
downward with an acceleration = 2.2 ms–2 and (iii) lift falls freely. Given g = 9.8 ms–2.

9. Consider the normal force on a book at rest on a table top. If the table is tilted so that the surface
forms an inclined plane, will the magnitude of the normal force change? Defend your answer

10. A force of gravity pulls downwards on a book on a table. What force prevents the book from
accelerating downward ?

77
Ka
vi
Class IX : Physics

ra
cl
as
se
l Statics is a branch of mechanics that deals with the forces acting on stationary objects. Dynamics

s
is the branch of mechanics that deals with the cause of motion i.e. forces acting on moving objects.

de
l A force exerted on an object can cause that object’s velocity to change, that is, a force can cause an

lh
acceleration.

i
l If you push a wall i.e. apply force on it, the wall will not move. This means practically, a force does
not always cause acceleration on an object.

l The net force acting on an object is also referred to as the total force, the resultant force, or the
unbalanced force acting on the object.

l An object is in equilibrium when it has zero acceleration. This means, it is the state of an object or
system of objects for which there are no changes in motion. It includes the state of rest as well as the
state of uniform motion.

l The equilibrium rule : For any object or system of objects in equilibrium, the sum of the forces
acting on it equals zero.

l If there is an unbalanced force acting on an object, the change in its velocity would continue as long
as this unbalanced force persists. If this force is removed completely, the object would continue to
move with the velocity it has acquired till then.

l If there is an unbalanced force acting on an object, its velocity changes, hence, its momentum also
changes. If the forces acting on an object are balanced, its velocity is constant, hence, its momentum is
also constant.

l In Newton’s second law, the force ‘F’ refers to the net external force (or unbalanced force) on the object
and ‘a’ refers to the acceleration of the object in the direction of net external force.
Dp
l Force, F = ; this means for a given change in momentum, the force is inversely proportional to
Dt
the time interval in which this change takes place. Thus, the same change in momentum brought
about in a shorter time needs a greater applied force, and vice-versa.

l Force is necessary for changing the direction of momentum, even if its magnitude is constant. We
can feel this while rotating a stone in a horizontal circle with uniform speed by means of a string.

l The total momentum of an isolated system of objects remains constant. This is an another
statement of the law of conservation of momentum.
D\Allen-Junior wing\Physics\IX\Unit-1\2-Force & Laws of Motion\2019-20

l Rocket propulsion or the recoil of gun are based on law of conservation of momentum as well as
Newton’s third law. This is because the law of conservation of momentum is derived using Newton’s
third law.

l Momentum is conserved for all types of collisions that take place in real world whenever external
forces don't interfere.

l It is important to note that even though the action and reaction forces are always equal in mag-
nitude, these forces may not produce accelerations of equal magnitudes. This is because each
force acts on a different object that may have a different mass.

78
Ka
vi
ra
Force and Laws of Motion

cl
as
1. When balanced forces act on a body, the body

se
(A) must remain in its state of rest if already at rest

s
(B) must continue moving with uniform velocity, if already in motion

de
(C) must experience some acceleration
(D) both (A) and (B)

lh
i
2. Which of the following statements is not correct for an object moving along a straight path in an
accelerated motion?
(A) Its speed keeps changing (B) Its velocity always changes
(C) It always goes away from the earth (D) A force is always acting on it

3. A passenger sitting in a train with his face in the direction of the moving train, tosses a coin which
falls behind him. It means that motion of the train is
(A) accelerated (B) uniform (C) retarded (D) along circular tracks

4. An object of mass 2 kg is sliding with a constant velocity of 4 m s–1 on a frictionless horizontal table.
The force required to keep the object moving with the same velocity is
(A) 32 N (B) 0 N (C) 2 N (D) 8 N

5. A rider on horse back falls when horse starts running all of a sudden because :
(A) Rider is taken back
(B) Rider is suddenly afraid of falling
(C) Inertia of rest keeps the upper part of body at rest where as lower part of the body moves
forward with the horse.
(D) Nothing can be said due to insufficient data.

6. Which of the following is not an example of newtons third law :


(A) Walking of a man
(B) Water moving out of a hose pipe, with large velocity
(C) Gun and bullet
(D) None of these

7. A book is lying on the surface of table as shown : N


W = weight of book & N = Normal reaction of table on book
(A) W and N are action-reaction pair
(B) W and N are not action-reaction pair W
(C) W and N may or may not be action-reaction pair
(D) None of them
D\Allen-Junior wing\Physics\IX\Unit-1\2-Force & Laws of Motion\2019-20

8. Match the following :


Column A Column B
(i) Friction (a) Perpendicular to surface.
(ii) Normal force (b) Book lying on a table.
(iii) Balanced force (c) A freely falling body.
(iv) Unbalanced force (d) Parallel to surface.
(A) (i) - (d); (ii) - (a); (iii) - (c); (iv) - (b) (B) (i) - (a); (ii) - (d); (iii) - (c); (iv) - (b)
(C) (i) - (a); (ii) - (d); (iii) - (b); (iv) - (c) (D) (i) - (d); (ii) - (a); (iii) - (b); (iv) - (c)

79
Ka
vi
Class IX : Physics

ra
9. In a game of tug of war, two persons pull on a horizontal rope from opposite sides. The winner will

cl
be the person who

as
(A) exerts greater force on the ground (B) exerts greater force on the rope

se
(C) exerts smaller force on the ground (D) exerts smaller force on the rope

s
10. An object of mass 10kg moves at a constant speed of 10m/s. A constant force, that acts on body

de
for 4s on the object, gives it a speed of 2m/s in the opposite direction. The force acting on the

lh
object is

i
(A) –3N (B) –30N (C) 3N (D) 30N

11. A body of mass 2kg moving on a horizontal surface with an initial velocity of 4ms–1 comes to rest
after 2s. If one wants to keep the body moving on same surface with velocity of 4ms–1, the force
required is
(A) zero (B) 4N (C) 8 N (D) 16 N

12. A player caught a cricket ball of mass 150g which came to his hand with a speed of 20m/s. If the
ball was stopped in 0.1s, the force exerted by ball on the hands of player is:
(A) 150N (B) 3kN (C) 30N (D) 15N

13. A machine gun fires n bullets per second and the mass of each bullet is m. If the speed of bullets is
v, then the force exerted on the machine gun is
mnv
(A) mng (B) mnv (C) mnvg (D)
g

14. A body of mass 5kg undergoes a change in speed from 20m/s to 0.20 m/s. The momentum :
(A) Increases by 99 kg m/s (B) Decreases by 99kg m/s
(C) Increases by 101 kg m/s (D) Decreases by 101 kg m/s

15. Two identical bodies are allowed to fall from different heights h1 & h2. The ratio of their momentum
when they reach the ground will be
(A) h1: h2 (B) h12 : h22 (C) h1 : h 2 (D) 1

16. A body P has mass 2 m and velocity 5 v. Another body Q has mass 8 m and velocity 1.25 v. The
ratio of momentum of P and Q is
(A) 2 : 1 (B) 1 : 1 (C) 1 : 2 (D) 3 : 2

17. A car of mass 200 kg is moving with a speed of 20 m/s, after 25 seconds the velocity increased by
10 m/s, then what is the change in momentum ?
(A) 4000 kg m/s (B) 3000 kg m/s (C) 1000 kg m/s (D) 2000 kg m/s
D\Allen-Junior wing\Physics\IX\Unit-1\2-Force & Laws of Motion\2019-20

18. A force-time graph for motion of a body is shown in figure. The change in momentum is :

F(N)

1 7
0 T(s)
–1 2 4 6 8
–2

(A) zero (B) 4 Ns (C) 8 Ns (D) 16 Ns

80
Ka
vi
ra
Force and Laws of Motion
19. A body of mass 2 kg moving in opposite directions with a velocity of 3 m/s collides head on with a

cl
body of mass 1 kg moving with a velocity of 4 m/s. After collision the two bodies stick together and

as
move with a common velocity of :

se
(A) (1/4) m/s (B) (1/3) m/s (C) (2/3) m/s (D) (3/4) m/s

s
20. A bullet weighing 10 g and moving at 300 m/s strikes a 5 kg block of ice. The ice block is sitting on

de
frictionless level surface. The speed of the block, after the collision, is :

lh
(A) 6 cm sec–1 (B) 6 m sec–1 (C) 60 cm sec–1 (D) 60 m sec–1

i
21. A body of mass 2.5 kg takes 5 seconds to stop when a force 10 N is applied in a direction opposite
to the motion of the body. The initial speed of the body was :
(A) 20 m/s (B) 10 m/s (C) 15 m/s (D) 25 m/s

22. A particle at rest suddenly explodes into two fragments of equal masses, which starts moving. The
fragments will
(A) move in opposite directions with unequal velocities
(B) move in opposite directions with equal speeds
(C) move in any direction with equal speeds
(D) move in same direction with equal velocities

23. A bullet of mass m and velocity v is fired onto a stationary block of mass M and sticks to it. The
final velocity would be:
m M-m m m+M
(A) v (B) v (C) v (D) v
m+M M M-m M

24. If mass of body is increased by 10 % and its speed is decreased by 10 % then percentage change
in magnitude of its momentum will be :
(A) 1 % increase (B) 1 % decrease (C) 2 % increase (D) 2 % decrease

25. A bomb of 50 kg is fired from a cannon with a velocity + 600 m/s. If the mass of the cannon is
103 kg, then its recoil velocity will be :
(A) 30 m/s (B) –30 m/s (C) 0.30 m/s (D) –0.30 m/s
D\Allen-Junior wing\Physics\IX\Unit-1\2-Force & Laws of Motion\2019-20

81
Ka
vi
Class IX : Physics

ra
cl
as
se
1. When a motor car of mass 1500 kg is pushed on a road by two persons, it moves with a small
uniform velocity. On the other hand, if this car is pushed on the same road by three persons, it

s
moves with an acceleration of 0.2 m/s2. Assume that each person is producing the same muscular

de
force. Then, the force of friction between the tyres of the car and the surface of the road is :

lh
(A) 300N (B) 600N (C) 900N (D) 100N

i
2. The figure shows the variation of force acting on a body with time. Assuming the body to start from
rest, choose the best variation of its momentum with time :

F (N)

20

t (s)
0 2 4

p (kg ms-1) p (kg ms-1)

40 40

(A) 20 (B) 20

t (s) t (s)
0 2 4 0 2 4

p (kg ms -1) p (kg ms –1)

40

(C) 20 (D) 20

t (s) 0 2 4 t (s)
0 2 4
D\Allen-Junior wing\Physics\IX\Unit-1\2-Force & Laws of Motion\2019-20

3. A ball of mass m is dropped from a height h on a smooth elastic floor, such that it rebounds with
same speed, then, the change in magnitude of momentum of ball before and after striking the floor
is: (Take vertically downward direction as positive)
(A) 2m 2gh (B) -2m 2gh (C) m 2gh (D) zero

4. A body of mass ‘m’ kg starts from rest and travels a distance of ‘s’ m in ‘t’ seconds. The force acting
on it is

2ms ms ms 2 ms 2
(A) N (B) N (C) N (D) N
t2 t 2t t
82
Ka
vi
ra
Force and Laws of Motion
5. A missile of mass M moving horizontally with velocity v in free space explodes into two parts. After

cl
the explosion one of the parts of mass m falls vertically down. The other part proceeds with velocity

as
Mv Mv ( M - m) v mv

se
(A) (B) M - m (C) (D)
m ( ) M M-m

s
de
6. The ratio of weight of a man in a stationary lift and when it is moving downward with uniform
acceleration ‘a’ is 3 : 2. The value of ‘a’ is :

lh
g 3 2

i
(A) (B) g (C) g (D) g
3 2 3
7. The pulley arrangements of Fig. (A) and (B) are identical. The mass of the ropes is negligible. In (A),
the mass m is lifted up by attaching a mass 2m to the other end of the rope. In (B), m is lifted up by
pulling the other end of the rope with a constant downward force F = 2 mg. In which case the
acceleration of ‘m’ is more ?

a a'

a a'
m m
2m F = 2 mg

(A ) (B )

(A) B (B) A (C) Equal in both (D) Cannot be found

8. Two masses of 5 kg and 10 kg are connected to a pulley as shown. What will be the acceleration if
the pulley is set free ? (g = acceleration due to gravity)
(A) g
g
(B)
2
g
(C)
3
g
(D)
4
9. Three blocks are connected as shown in figure on a horizontal frictionless surface, tension T2 is :
D\Allen-Junior wing\Physics\IX\Unit-1\2-Force & Laws of Motion\2019-20

T1 8kg T2 27kg
1kg 36N

(A) 18 N (B) 9 N (C) 3 N (D) 1.5 N

10. Two blocks are in contact on a frictionless table. One has mass m and the
other 2m. A force F is applied on 2 m as shown in figure. Now the same
force F is applied from the right on m. In the two cases the ratio of force of
contact between the two blocks will be
(A) 1 : 1 (B) 1 : 2 (C) 2 : 1 (D) 1 : 3

83
Ka
vi
Class IX : Physics

ra
cl
as
Very short answer type questions

se
1. On what factor does inertia of a body depend?

s
2. State Newton’s second law of motion.

de
lh
3. State law of conservation of momentum.

i
4. An aeroplane flies at constant speed in a horizontal straight path. Two horizontal forces act on the
plane. One is the thrust of the propellers that pushes it forward, and the other is the force of air
resistance that acts in the opposite direction. Which force is greater?

5. Name the physical quantity which corresponds to rate of change of momentum ? (CBSE 2011)

Short answer type questions


6. There are three solids made up of aluminium, steel and wood, of the same shape and same volume.
Which of them would have highest inertia?

7. A horse continues to apply a force in order to move a cart with a constant speed. Explain why ?

8. Suppose a ball of mass m is thrown vertically upward with an initial speed v; its speed decreases
continuously till it becomes zero. Thereafter, the ball begins to fall downward and attains the speed
v again before striking the ground. It implies that the magnitudes of initial and final momenta of
the ball are same. Yet, it is not an example of conservation of momentum. Explain why?

11. (a) State Newton‘s third law of motion. (CBSE 2011)


(b) Why does a boat move backward, when a man jumps out of a boat to the bank of a river ?

Long answer type questions


10. Derive the mathematical form of second law of motion. A force of 5 N produces an acceleration
of 8 m s–2 on a mass m1 and an acceleration of 24 m s–2 on a mass m2. What acceleration would
the same force provide if both the masses are tied together?

11. (a) State the law of conservation of momentum (CBSE 2011)


(b) During collision of two balls total momentum of the system remains unchanged? Justify this
statement mathematically.
(c) Why does a gun recoil when a bullet is fired from it ? Explain.

12. (a) State the Newton‘s second law of motion. (CBSE 2011)
(b) Express Newton‘s second law mathematically explaining the symbols used.
(c) Define SI unit of force from this expression
D\Allen-Junior wing\Physics\IX\Unit-1\2-Force & Laws of Motion\2019-20

Numerical problems
13. A constant retarding force of 50 N is applied to a body of mass 20 kg moving initially with a speed
of 15 m s–1. How long does the body take to stop?

14. The driver of a three-wheeler moving with a speed of 36 km/h sees a child standing in the middle
of the road and brings his vehicle to rest in 4.0 s just in time to save the child. What is the average
retarding force on the vehicle ? The mass of the three-wheeler is 400 kg and the mass of the driver
is 65 kg.

15. A shell of mass 0.020 kg is fired by a gun of mass 100 kg. If the muzzle speed of the shell is 80
m s–1, what is the recoil speed of the gun?

84
Ka
vi
ra
Force and Laws of Motion

cl
as
1. The maximum and minimum magnitudes of the resultant of two forces are 17 newton and 7 newton

se
respectively, If these two forces are acting at right angle to each other, then the value of resultant of

s
these forces in newton will be : (Rajasthan/NTSE Stage-I 2006)

de
(A) 10 (B) 12 (C) 13 (D) 24

lh
2. In a very heavy lorry moving on the road with slightly falttened tyres [INJSO 2009]

i
(A) only rolling friction is involved.
(B) both rolling and kinetic friction are involved.
(C) only kinetic friction is involved.
(D) The type of friction depends on the speed of the lorry.

3. A soldier with a machine gun, falling from an airplane gets detached from his parachute. He is able
to resist the downward acceleration if he shoots 40 bullets a second at the speed of 500 m/s. If the
weight of a bullet is 49 gm, what is the weight of the man with gun? Ignore resistance due to air and
assume the acceleration due to gravity g = 9.8 m/s2. [KVPY 2010]
(A) 50 kg (B) 75 kg (C) 100 kg (D) 125 kg

4. When a constant force is applied to a body, it moves with uniform


(Rajasthan/NTSE Stage - I 2011)
(A) Speed (B) velocity (C) Acceleration (D) Momentum

5. When a car turns on a curved road, you are pushed against one of the doors of the car because of
[NSEJS, 2012]
(A) inertia (B) the centripetal force
(C) the centrifugal force (D) the frictional force

6. What is the reading of the spring balance shown in figure below? [NSEJS, 2012]
T T

2kg

(A) 0 (B) 2 kgf (C) 4 kgf (D) 6 kgf

7. A certain force applied to a body A gives it an acceleration of 10 ms–2. The same force applied to
body B gives it an acceleration of 15 ms–2. If the two bodies are joined together and the same force
D\Allen-Junior wing\Physics\IX\Unit-1\2-Force & Laws of Motion\2019-20

is applied to the combination, the acceleration will be [NSEJS, 2012]


(A) 6 ms–2 (B) 25 ms–2 (C) 12.5 ms–2 (D) 9 ms–2

8. A ball is dropped from a height of 7.2 m. It bounces back to 3.2 m after striking the floor. The ball
remains in contact with the floor for 20 ms. Given that g = 10 ms–2, the average acceleration of the
ball during the contact is [NSEJS, 2012]
(A) 100 ms–2 (B) 200 ms–2 (C) 600 ms–2 (D) 1000 ms–2

9. A Parachutist of weight W strikes the ground with his legg fixed and come to rest with an upward
acceleration of magnitude 3g. Force exerted on him by the ground during landing is
(Delhi/NTSE Stage - I 2012)
(A) W (B) 2W (C) 3W (D) None of these
85
Ka
vi
Class IX : Physics

ra
10. Unit of Impulse is (Madhya Pradesh/NTSE Stage-I 2013)

cl
(A) Newton (B) Newton × Second

as
(C) Newton × Second2 (D) Newton / Second

se
s
11. A boy sitting on the top most berth in the compartment of a train which is just going to stop on the
railway station, drops an apple aiming at the open hand of his brother situated vertically below his

de
hands at a distance of about 2m. The apple will fall. (Punjab/NTSE Stage - I 2013)

lh
(A) In the hand of his brother

i
(B) Slightly away from the hand of his brother in the direction of the motion of the train.
(C) Slightly away from the hand of his brother in the direction opposite to the direction of the
motion of the train.
(D) None of these

12. An object is moving with a speed of 1.0 m/s. A force F 1 is required to stop it over a distance X. If the
speed of the object increases to 3.0 m/s, a force F2 is required to stop it over the same distance X.
F1 : F2 will be (Chandigarh/NTSE Stage - I 2013)
(A) 1 : 3 (B) 3 : 1 (C) 1 : 9 (D) 1 : 6

13. Which important law is directing towards Newton's second and third law of motion ?
(Gujarat/NTSE Stage - I 2013)
(A) Law of Pressure (B) Law of Conservation of Momentum
(C) Principle of Archimedes (D) Law of Energy Conservation

14. If unbalanced force applies on object, then object......... (Gujarat/NTSE Stage - I 2013)
(A) remains stable (B) moving with acceleration
(C) moving with constant velocity (D) moving with circular motion
15. A block is placed on a rough horizontal surface. A time dependent horizontal force F = kt acts on
the block, where k is positive constant. Acceleration time graph of the block is
(Haryana/NTSE Stage - I 2013)

a a

(A) (B)

t t

a a
D\Allen-Junior wing\Physics\IX\Unit-1\2-Force & Laws of Motion\2019-20

(C) (D)

t t

16. Which is not the unit of Force? (Rajasthan/NTSE Stage - I 2013)


(A) Poundal (B) Dyne
(C) Joule (D) Newton
17. Two bodies of masses m and 3m, moving with velocities 3v and v respectively along same direction,
collide with each other. After collision they stick together and move with a velocity V in the same
direction. Then: (Orissa/NTSE Stage-I 2013)
3 4
(A) V = v (B) V = v (C) V = 2v (D) V = v
2 3
86
Ka
vi
ra
Force and Laws of Motion
18. A single horizontal force F is applied to a block of mass M1 which is in contact with another block

cl
of mass M2 as shown in the figure. If the surfaces are frictionless, the force between the block is:

as
(West Begal/NTSE Stage-I 2013)

se
s
F M1 M2

de
M1F M1M 2F M 2F M 2F

lh
(A) M (B) M + M (C) M + M (D) M
2 1 2 1 2 1

i
19. In the figure, the blocks ABC of mass m each have acceelrations a1, a2 and a3 respectively. "F1" and
"F2" are external forces of magnitude 2mg and mg respectively. (Haryana/NTSE Stage-I 2013)

m m m
A B C
F1=2mg 2m m

F2=mg
(A) a1 = a2 = a3 (B) a1 > a3 > a2 (C) a1 > a2 > a3 (D) a1 > a2, a2 = a3

20. The figure given below shows the displacement plotted time for a particle. In which regions is the
force acting on the particle zero ? (Bihar/NTSE Stage - I 2014)
Displacement

B C E
D
A
Time
(A) AB (B) BC (C) CD (D) DE

21. The elongation of wire of length L is l in the case of figure (i). The same wire elongation in case of
figure (ii) will be (pulley is light) (Haryana/NTSE Stage-I 2014)

M M M
fig(i) fig(ii)
D\Allen-Junior wing\Physics\IX\Unit-1\2-Force & Laws of Motion\2019-20

(A) 4l (B) 2l (C) l (D) l/2

22. According to Newton's Second law of motion- (Madhya Pradesh/NTSE Stage-I 2014)
m m
(A) f = m × v (B) f = m × a (C) f = (D) f =
a v
23. A body of mass 2 kg is moving on a smooth floor in straight line with a uniform velocity of 10 m/s.
Resultant force acting on the body is: (Rajasthan/NTSE Stage - I 2015)
(A) 20 N (B) 10 N (C) 2 N (D) Zero

24. Which law of newton gives the definition of force ? (Gujrat/NTSE Stage-I 2015)
(A) First law (B) Second law (C) Third law (D) Fourth law
87
Ka
vi
Class IX : Physics

ra
25. A wooden block (W) is suspended by using a cord from a heavy steel ball (B). The entire system

cl
is dropped from a height.Neglecting air resistance, the tension in the cord is (NSEJS, 2016)

as
(A) Zero

se
(B) The difference in the masses of B and W
(C) The differences in the weights of B and W

s
(D) The weight of B

de
lh
26. Velocity of particle moving along a straight line varies with time as shown in the adjacent figure.
Net forces acting on the particle are F1, F2, F3, F4 and F5 in the intervals OA, AB, BC, CD and

i
DE respectively. Identify the correct statement– v (NSEJS, 2016)
(A) F1 increases with time A
D
(B) F5 is intially positive and then becomes negative
O t
(C) F1 and F2 are in opposite directions
B C E
(D) F3 is negative

27. A bullet of mass 10 gm moving with 100 m/s is embedded in a block of 1 kg which is initially in rest.
The final velocity of the system will be– (Rajasthan/NTSE Stage-I 2016)
(A) 1 m/s (B) 1.5 m/s (C) 0.5 m/s (D) 2 m/s

28. The momentum of an object at a given instant is independent of its:


(Rajasthan/NTSE Stage-I 2016)
(A) Inertia (B) Speed (C) Velocity (D) Acceleration

29. The pressure exerted on the ground by a man is greatest when:


(Rajasthan/NTSE Stage-I 2016)
(A) He stands with both feet flat on ground (B) He Stands flat on one foot
(C) He stands on the toes of one foot (D) All the above yield the same pressure

30. A body of mass 2 kg is free to move along x-axis. It is at rest at t = 0. From t = 0 onward it is
subjected to a time-dependent force F(t) in x-direction. The variation of F with time is as shown.
The change in momentum during journey is (in N s). (Haryana/NTSE Stage-I 2016)
(A) 4.50 F(t)
4N
(B) 7.50

(C) 5.06 4.5s t


O 3s
(D) 14.06 –2N
31. A horizontal jet of water is made to hit a vertical wall with a negligible rebound. If the speed of water
from the jet is ‘v’, the diameter of the jet is ‘d’ and the density of water is ‘r’, then the force exerted
D\Allen-Junior wing\Physics\IX\Unit-1\2-Force & Laws of Motion\2019-20

on the wall by the jet of water is- (NTSE Stage-II 2017)


p 2 p 2 2 p 2 2 p 2 2
(A) d ru (B) d ru (C) d ru (D) d ru
4 4 8 2

32. Two blocks A and B of masses 8 kg and 2 kg respectively,


lie on a horizontal frictionless surface as shown in the
figure. They are pushed by a horizontally applied force of
15 N. The force exerted by B on A is A
15 N 8 kg B
(NTSE Stage-II 2017)
(A) 1.5 N (B) 3.0 N 2 kg
(C) 4.5 N (D) 6.0 N
88
Ka
vi
ra
Force and Laws of Motion
33. A bullet of mass 50 gm is horizontally fired with a velocity 100 ms –1 from a gun of mass 10 kg. What

cl
will be the recoil velocity of the gun ? (Rajasthan / NTSE Stage-I 2017)

as
(A) 100 ms–1 (B) 500 ms–1 (C) 0.5 ms–1 (D) Zero

se
34. An empty office chair is at rest on a floor. Consider the following forces : [NSEJS, 2017]

s
I- A downward force of gravity,

de
II-An upward force exerted by the floor,

lh
III-A net downward force exerted by the air.

i
Then, which of the force(s) is (are) acting on the office chair?
(A) I only (B) I and II
(C) I, II and III
(D) None of the forces. (Since the chair is at rest there are no forces acting upon it.)

35. A large truck collides head-on with a small compact car. During the collision : [NSEJS, 2017]
(A) the truck exerts a greater force on the car than the car exerts on the truck.
(B) the car exerts a greater force on the truck than the truck exerts on the car.
(C) the truck exerts a force on the car but the car does not exert a force on the truck.
(D) the truck exerts the same force on the car as the car exerts on the truck.

36. If a body exerts a force on a surface, the surface exerts a reaction force on the body. This reaction
force is always along normal to the surface at that point and is popularly known as the normal force
N. When the body slides on a surface, the surface exerts a force of friction on the body in the
direction opposite to the direction of slipping. The magnitude of this frictional force is given by mN,
where m is a constant (called coefficient of friction) for the given pair of surfaces in contact and N
is the normal force.
Consider a block of mass 100 kg placed on a level surface. If one person tries to push it with a force
F, he is unable to do so. He receives help from two of his friends and each one applies the same
force F. Now the block moves with a uniform acceleration of 0.5 m/s2. After sometime, one of them
happens to leave the exercise of pushing the block. This time the block moves with a uniform speed.
The magnitude of force F and the coefficient of kinetic friction m are respectively____________.
[INJSO, 2018]
(A) 50 N, 0.05 (B) 100 N, 0.1 (C) 50 N, 0.1 (D) 100 N, 0.2

37. The inertia of a body depends upon (Rajasthan / NTSE Stage-I 2018)
(A) Gravitational acceleration (B) Centre of gravity of body
(C) Shape of body (D) Mass of body

38 Suppose our scientific community had chosen force, speed and time as the fundamental mechanical
quantities instead of length, mass and time respectively and they chose the respective units of
1
D\Allen-Junior wing\Physics\IX\Unit-1\2-Force & Laws of Motion\2019-20

magnitudes 10 N, 100 m/s and s. Then the unit of mass in their system is equivalent to _______
100
in our system. [NSEJS, 2018]
(A) 10 kg (B) 10 kg
–3
(C) 10 kg (D) 10 kg
–1

39. Four blocks of different masses (m1 = 1 kg, m2 = 2kg, m3 = 1 kg and m4 = 5 kg) ae connected with
light, inextensible strings, as shown in figure. This system is pulled along a frictionless surface by a
horizontal force of 36N. The force pulling the block of mass m1 will be: (NTSE Stage-II 2018)
m4
m1 m2 m3
2 5 36 N
1 1

(A) 2 N (B) 4 N (C) 12 N (D) 36 N


89
Ka
vi
Class IX : Physics

ra
ANSWERS

cl
as
CHECK POST-1

se
1. (A) 2. (A) 3. (A) 4. (B) 5. (D)

s
de
CHECK POST-2

lh
1. (C) 2. (B) 3. (D) 4. (C) 5. (A) 10. 120 m/s

i
CHECK POST-3
1. (D) 2. (A) 3. (A) 4. (D) 5. (A) 8. –40 dyne

CHECK POST-4
1. (B) 2. (A) 3. (B) 4. (B) 5. (D)
10. Players move with velocity of 0.26 m/s along direction of motion of second hockey player.

CHECK POST-5
1. (A) 2. (D) 3. (C) 4. (A) 5. (C) 6. a = 2 m/s2, F = 60 N
7. a = 2.5 m/s2, T = 5N 8. (i) 360 N, (ii) 228 N, (iii) Zero

EXERCISE-1 (ELEMENTARY)
Que. 1 2 3 4 5 6 7 8 9 10 11 12 13 14 15
Ans. D C A B C D B D A B B C B B C
Que. 16 17 18 19 20 21 22 23 24 25
Ans. B D A C C A B A B B

EXERCISE-2 (SEASONED)
Que. 1 2 3 4 5 6 7 8 9 10
Ans B B D A B A A C B B

EXERCISE-3 (CBSE PATTERN)

10. 6ms-2 13. 6s, 45 m 14. 1162.5 N 15. 1.6×10–2 m/s


D\Allen-Junior wing\Physics\IX\Unit-1\2-Force & Laws of Motion\2019-20

EXERCISE-4 : (COMPETITIVE ASSESSMENT)


Que. 1 2 3 4 5 6 7 8 9 10
Ans. C B C C A B A D D B
Que. 11 12 13 14 15 16 17 18 19 20
Ans. B C B B D C B C B A
Que. 21 22 23 24 25 26 27 28 29 30
Ans. B B D A A C A D C A
Que. 31 32 33 34 35 36 37 38 39
Ans. B B C B D C D B B

******
90

You might also like